Você está na página 1de 190

CrackVerbal GMAT Critical Reasoning Guide|i

GMAT Critical Reasoning

Guide

gmat.crackverbal.com
CrackVerbal GMAT Critical Reasoning Guide|ii

All Rights Reserved


© CrackVerbal Edutech Pvt. Ltd., 2017
ISBN 978-81-928450-4-3

gmat.crackverbal.com
CrackVerbal GMAT Critical Reasoning Guide|iii

Copyright Notice

As of 2017, CrackVerbal Edutech Pvt. Ltd. is the copyright holder of this study
material. It is under our discretion to demand consideration in exchange for waiver of
any of the conditions listed above. Where the study material or any of its content is
already in the public domain under law currently applicable in India or any location
where the study material is used, that status is in no way affected by the license.

Legal Disclaimer

The information contained in these slides is for general information purposes only.
We endeavor to keep the information up-to-date and correct, however, we make no
representations or warranties of any kind, express or implied, about the
completeness, accuracy, reliability, suitability or availability with respect to the slides
or the information, products, services, or related graphics contained on the slides for
any purpose. Any reliance you place on such information is, therefore, strictly at your
own risk.

In no event will CrackVerbal be liable for any loss or damage including without
limitation, indirect or consequential loss or damage, or any loss or damage
whatsoever arising from loss of data or profits arising out of, or in connection with,
the use of this material.

Through this material you may be able to link to other websites which are not under
the control of the owner of the material. The owner has no control over the nature,
content and availability of those sites. The inclusion of any links does not necessarily
imply a recommendation or endorse the views expressed within them.

GMAT™ and GMAC™ are registered trademarks of the Graduate Management


Admission Council™. GMAC does not write, sponsor, or endorse this product, nor is it
affiliated in any way with the owner of this material.

gmat.crackverbal.com
CrackVerbal GMAT Critical Reasoning Guide|iv

Foreword

Hello!

Thank you for picking up this copy of the CrackVerbal GMAT Critical Reasoning Guide.

In the past few years, CrackVerbal has helped 5000+ GMAT aspirants from all over the
world. Our techniques are built on the premise that Indians think very differently from
native speakers of English. So, in order to ace the GMAT, quintessentially an American
test, we need to think differently.

Let’s take the case of Critical Reasoning (CR) on the GMAT. This section requires you to
understand and analyze an argument, and answer questions based on the
assumptions, content, and structure of the argument. Though the term “reasoning”
may lead you to assume that you can rely only on your logic to crack CR questions,
doing so would be a mistake. If you rely only on common sense, your hit rate for
questions will be very erratic. On a good day, you may get quite a few CR questions
right. On a bad day, you may not!

The techniques discussed in this book are specifically designed for Indian GMAT
aspirants. They have been developed after a decade of teaching and research, and
have been revised many times to meet the changing GMAT trends.

Our focus in GMAT Critical Reasoning is to take away the whole theory and focus more
on the building blocks of logic, and to focus on a specific, structured approach for
every question type and trap in GMAT CR. Apart from concepts and strategies, we
have included in this book Cracker Tips for every possible topic and practice
questions to help you reinforce the concepts learned.

Hope this guide helps you with your GMAT preparation. All the very best!

The CrackVerbal Team

gmat.crackverbal.com
CrackVerbal GMAT Critical Reasoning Guide|v

Your Online Resources

YOUR PURCHASE INCLUDE ONLINE ACCESS TO THE FOLLOWING

E-books

• Frequently Tested Patterns on GMAT® Critical Reasoning :


This e-book introduces you to some commonly tested
patterns and traps used in GMAT Critical Reasoning. Using
examples to illustrate various strategies and common
traps, this book helps you gain an edge in test preparation
and should serve as an invaluable resource for your GMAT
Preparation.
• RC on GMAT : This guide discusses how to score well on GMAT Reading
Comprehension, using our famous ‘Mapping Technique’.
• 18 Grammar Concepts on GMAT: This book will help you brush-up your basic
grammar, especially those required to ace the Sentence Correction section on the
GMAT.
• 100 Tough GMAT® Questions with Solutions : This book is an excellent resource to
practice. It consists of 100 Tough questions with 20 Questions each of CR , SC , RC ,
PS and DS.

GMAT Foundation Course

1. GMAT Foundation Course : You will get access to our GMAT


Foundation Course , which will help you to build foundation
for GMAT. The course consists of 5 + Hrs of Videos

v Please find the Access code at the end of this book to access all online
resources for free.

gmat.crackverbal.com
CrackVerbal GMAT Critical Reasoning Guide|vi

How to Access the Online Resources

STEP 1: Go to https://goo.gl/8LJXwE

STEP 2: Click on enrol in the course and fill your details

STEP 3: After signing up you have to make payment.

STEP 4: Fill the Access Code provided to you in the coupon code.

STEP 5: After completing all steps Log In again and Access the Course & E-books

v Your Online Access will be valid till 1 Year from the Day you use your Access
Code.

v Please note that Online Access is Non-Transferrable , which means only New and
Unregistered copies of the Book will grant you access to online resources

gmat.crackverbal.com
CrackVerbal GMAT Critical Reasoning Guide|vii

Why Study With CrackVerbal?

India’s best GMAT trainers

Extensive Support & Mentorship

Comprehensive Study Material

Students-only Discussion Forum

Customized Study Plans

Want to get the CrackVerbal Advantage?

Just call +91 901-9199-800 OR +91 900-8177-800

Or write to enquiry@crackverbal.com

gmat.crackverbal.com
CrackVerbal GMAT Critical Reasoning Guide|viii

gmat.crackverbal.com
CrackVerbal GMAT Critical Reasoning Guide|ix

TABLE OF CONTENTS

1. Introduction to Critical Reasoning 3

Anatomy of a GMAT Critical Reasoning Question 5

3 Broad Types of CR Questions that appear on the GMAT 7

2. Assumption-Based CR Questions 9

Introduction to Assumption-Based CR Questions 11

Structure of an Assumption-Based CR question 13

Truth versus Validity of an Argument 14

Recognizing the Premises and Conclusion in an Argument 15

Type 1: Find the Assumption Questions 26

The Negation Technique 27

CrackVerbal’s 5-Step Approach to Tackle CR on the GMAT 29

3. Five Common Traps in Assumption-based Questions 31

Causality versus Correlation 33

Comparison 36

Statistical Traps 37

Harm/Benefit 40

Language Shift 41

Identifying Structural Patterns in GMAT CR 41

gmat.crackverbal.com
CrackVerbal GMAT Critical Reasoning Guide|x

4. Strengthen/Weaken the Conclusion 63

Type 2: Strengthen the Conclusion 65

Type 3: Weaken the Conclusion 67

Out of Scope answer choices 69

Except Questions 89

5. Flaw & Evaluate Questions 91

Type 4: Flaw in the Argument 93

Type 5: Evaluate the Argument 94

6. Content-Based Questions 113

How are Content-Based Questions Different

From Assumption Based Questions? 115

Type 1: Inference/Conclusion/Main Point Questions 116

4 Golden Rules for Content-based Questions 116

Type 2: Resolve the Paradox 134

7. Structure-Based Questions 147

Type 1: Bold-Faced Questions 149

Step-by-Step Approach for Bold-Faced Questions 151

Terminology commonly used in bold-faced questions 153

Type 2: Parallel or Mimic the Reasoning 162

gmat.crackverbal.com
CrackVerbal GMAT Critical Reasoning Guide|xi

8. Advanced test taking strategies for CR 173

Practice Questions

Find the Assumptions 43

Strengthen and Weaken 73

Flaw and Evaluate 97

Inference / main point / conclusion 119

Resolve the paradox 137

Bold Faced type 154

Mimic the reasoning questions 165

CR Mind Map 177

gmat.crackverbal.com
CrackVerbal Critical Reasoning Guide|1

gmat.crackverbal.com
CrackVerbal Critical Reasoning Guide|2

gmat.crackverbal.com
CrackVerbal Critical Reasoning Guide|3

1: Introduction to Critical Reasoning

• Anatomy of GMAT Critical Reasoning Question


• 3 Broad Buckets of Critical Reasoning Questions

gmat.crackverbal.com
CrackVerbal Critical Reasoning Guide|4

gmat.crackverbal.com
CrackVerbal Critical Reasoning Guide|5

Critical Reasoning on the GMAT tests your understanding of the


construction, structure, and components of logical arguments as well as your
ability to carefully read and interpret the English language. In normal life we
tend to argue using common sense, while on the GMAT, you are expected to use
only logic.

Typically, there would be about 12-14 Critical Reasoning questions in the


GMAT Verbal section, out of a total of 41 questions. Each CR question will
include a stimulus which can range from 16 to 125 words. This will be followed
by a question stem, usually about 1 or 2 lines long. 5 answer choices will be
provided out of which you need to pick one correct option.

Questions will appear in order of difficulty, based on how well (or poorly!)
you are doing on the exam.

ANATOMY OF A GMAT CRITICAL REASONING QUESTION

Directions: Analyze the situation on which each question is based, and then,
select the answer choice that is the most appropriate response to the question.
No specialized knowledge of any particular field is required for answering the
questions, and no knowledge of the terminologies and conventions of formal
logic are presupposed.

Although the directions mention that ‘no knowledge of the terminology and
conventions of formal logic is presupposed’, it doesn’t hurt to have some! So,
we will later cover some background in formal logic.

gmat.crackverbal.com
CrackVerbal Critical Reasoning Guide|6

Stimulus:

The Spanish mackerel and other commonly eaten fish are at a higher risk of
contracting fungal infection when industrial effluents contaminate their water.
A proposal has been put forth to reroute offshore, gallons of industrial efflu-
ents every day. Although this would substantially reduce the amount of efflu-
ents in the water bodies where mackerel are caught, the proposal is pointless,
because hardly any mackerels live long enough to be harmed by these infec-
tions.

Question:

Which one of the following, if true, most seriously weakens the argument?

Options:

A. Contaminants in the water other than industrial effluents are equally harmful
to mackerel.
B. Humans often become ill as a result of eating mackerel with fungal infec-
tions.
C. Mackerel, like other fresh-water fish, live longer in the isolated lakes than in
water bodies close to industrialized land.
D. Mackerel breed as readily in effluent-contaminated water as in unpolluted
water.
E. Fungal infections cannot be detected by examining the surface skin or scales
of the fish.

The point of the illustrative example is to show you the different parts of a
CR question. But if you are curious, this is a ‘weaken the argument’ question; it
requires understanding the assumption of the argument. The answer, here, is B.

The trick to crack CR questions on the GMAT is to see beyond the topic/content
and analyze the structural relationships present in the question. We will
explore how to do this in upcoming chapters of this book. We will be discussing
strategies on how to deal with such questions in the following chapters.
gmat.crackverbal.com
CrackVerbal Critical Reasoning Guide|7

3 BROAD BUCKETS OF CRITICAL REASONING QUESTIONS

There are three broad buckets in which CR questions are tested on the
GMAT. They are as follows.

1. Assumption based questions


i. Find the assumption questions
ii. Strengthen questions
iii. Weaken questions 60%
iv. Find the flaw questions
v. Evaluate the argument questions

2. Content based questions


i. Inference questions 30%
ii. Resolve the paradox

3. Structure based questions


i. Bold faced questions 10%
ii. Flaw questions

The above percentages are rough indicators

gmat.crackverbal.com
CrackVerbal Critical Reasoning Guide|8

gmat.crackverbal.com
CrackVerbal Critical Reasoning Guide|9

2. Assumption Based CR Questions

• Introduction to Assumption-Based CR Questions


• Structure of an Assumption-Based CR Question
• Truth versus Validity of an Argument
• Recognizing Premises and Conclusion in an Argument
• Identifying Assumptions.
• Identifying Structural Patterns in GMAT CR
• The 5-Step Approach to Tackle CR on the GMAT
• Causality in GMAT CR

gmat.crackverbal.com
CrackVerbal Critical Reasoning Guide|10

gmat.crackverbal.com
CrackVerbal Critical Reasoning Guide|11

INTRODUCTION TO ASSUMPTIONS BASED CR QUESTIONS

Let us try to understand the various parts of an assumption-based CR question


and the relationships between them through some examples.

Situation

Mike and Molly are engaged to be married. One day Mike gets to know that
Molly went to the movies without him. This news disturbed Mike greatly. He
decided that Molly didn’t love him anymore.

You are Mike’s friend. What would you tell him?

If you are a good friend, you would probably calm him down and tell him not to
read too much into this piece of information. So what if Molly went to the
movies without him? It doesn’t necessarily mean that she has stopped loving
him. You would probably list out many arguments as to why Mike’s conclusion
isn’t justified.

Now let’s try to analyze Mike’s argument.

What is the information given in the argument?

• The relationship between Mike and Molly

• The fact that Molly went to a movie without Mike

These are called premises. We will look at these in more detail later.

What is Mike concluding on the basis of these premises?

He is concluding that Molly does not love him anymore.

gmat.crackverbal.com
CrackVerbal Critical Reasoning Guide|12

Thus, this is the conclusion.

What would you argue, as a friend, to console Mike?

Some pointers:

Did Molly go to the movie with her family or girl-friends?


If she did, then Mike has nothing to worry about.

Did Molly think that Mike was out-of-station and hence, did not inform him?
If true, this would explain why she didn’t let him know her plans.

For Molly, does going to a movie with someone qualify as a romantic gesture?
If Molly doesn’t think of going to the movies as a romantic experience, then
irrespective of who she goes with, Mike needn’t worry.

Is the movie of a genre that Mike dislikes and would normally not watch?
This is another reason why she wouldn’t have invited him along.

The answers to these questions will determine whether Mike’s conclusion is


sound. Thus, the truth of the premises alone is not sufficient to prove the
conclusion beyond any possible doubt. The answers to the above questions,
which determine the soundness of Mike’s conclusion, are called the assump-
tions.

Assumptions are unstated premises which determine whether an argument is


valid. They have dual nature, i.e., they may be true or false. The truth of as-
sumptions determines the validity of the argument.

Assumptions must necessarily be true for the argument to be valid.

gmat.crackverbal.com
CrackVerbal Critical Reasoning Guide|13

Let’s see whether you are able to analyze each of these arguments. Do you
think they are sound? Why or why not?

1. Apple doesn’t spend a lot on marketing its products; yet, its products are a
huge success with consumers world-wide. It is a $300bn company today.
Clearly, Apple has got the magic sauce to success —never market heavily if you
want to succeed.

2. The sun rises just a few minutes after Jake wakes up. This happens every day.
Clearly, Jake is causing the sun to rise.

STRUCTURE OF ASSUMPTION-BASED CR QUESTIONS

In this section, we will dissect the structure of an assumption-based CR


question and understand its parts. Assumption-based questions on the GMAT
have the same basic structure consisting of 3 parts:
1. Premise(s)
2. Assumption
3. Conclusion
1. Premise:

These are the evidences, reasons, facts, or statistics provided by the


author to support the conclusion. Premises have to be taken for granted, irre-
spective of whether they make intuitive sense to you or not.
2. Assumption:

This is the unstated part of the argument that is required to link the prem-
ises and the conclusion. An assumption is never explicitly mentioned in the
argument and has to be identified.

An argument can have more than one premise and more than one assump-
tion. But it can never have more than one conclusion.

gmat.crackverbal.com
CrackVerbal Critical Reasoning Guide|14

3. Conclusion:

This is the author’s point of view or the claim made by the author. This is
what needs to be proved or disproved.

TRUTH VERSUS VALIDITY OF AN ARGUMENT

Now that we have understood the structure and parts of an argument, let
us look at the reasonableness or the soundness of the argument. There are 2
concepts you need to understand first: truth and validity.

Prima facie, these may seem to indicate the same thing, but in critical
reasoning, these two term mean different things. Let us understand this using
an example:

All men are donkeys.


All donkeys have tails.
Therefore, all men have tails.

Is this argument valid?

Yes. If the first two statements (premises) are true, then the third statement
(conclusion) is also true. Thus, the argument is perfectly valid.

However, is this argument necessarily true? That is, based on our intuitive,
real-world knowledge, would we agree that all men are donkeys and therefore,
have tails? Absolutely not! Thus, this argument is not true.

On the GMAT, we are concerned only with the validity of arguments and not
their inherent truth. So if a GMAT question tells you that a country’s economy is
booming, but you know it is not, you still have to take the GMAT’s word for it!

gmat.crackverbal.com
CrackVerbal Critical Reasoning Guide|15

RECOGNIZING THE PREMISES AND CONCLUSION IN AN ARGUMENT

Here is the recommended 2 approaches to understanding and breaking


down every CR question on the GMAT.

1: One Sentence Summary

For every CR question, try to summarize the main point of the argument that
the author is trying to convey, in a single sentence This does not mean that you
have to summarize the entire logic of the argument in one sentence.
Example:

Doctor: As established by many research studies, adolescents who play video


games on a regular basis are three times as likely to develop carpal tunnel
syndrome as are adolescents who do not play video games. Therefore, federal
legislation that strictly prohibits the sale of video games to minors would help
curb this painful wrist condition among adolescents.

How would you describe this in one sentence?

Stopping minors from buying Video Games will reduce incidence of CTS among
them.

2: Keywords Identification

In every CR question, look out for certain keywords that indicate which lines
form the premise and which ones form the conclusion/counter-conclusion.
Don’t bother memorizing these words though; instead, get into the habit of
looking for such words that indicate the role a sentence plays in the argument.

gmat.crackverbal.com
CrackVerbal Critical Reasoning Guide|16

Premise Indicators:

• Because
• Since
• For
• For example
• For the reason that
• In that
• Given that
• As indicated by/due to
• Owing to
• This can be seen from/we know this by

Conclusion Indicators:

• Thus
• Therefore
• Hence
• Consequently
• As a result
• So
• Accordingly
• Clearly
• Must be that
• Shows that
• Concludes that

gmat.crackverbal.com
CrackVerbal Critical Reasoning Guide|17

Additional Premise Indicators:

Additional premises give additional supporting evidence to the conclusion;


they help build the argument. A few words that indicate additional premises
are

• Furthermore
• Moreover
• Besides
• In addition
• What’s more

Counter-Premise Indicators:

These are data points that support an opinion that the main argument goes
against or challenges. These play a huge role in helping us understand the
structure of the argument (more about this when we discuss structure based
questions). Some words that indicate counter-premises are-

• But
• Yet
• However
• On the other hand
• Admittedly
• In contrast
• Although
• Even though
• Still
• Whereas
• In spite of
• Despite
• After all

gmat.crackverbal.com
CrackVerbal Critical Reasoning Guide|18

Example 1:
Doctor: As established by many research studies, adolescents who play video
games on a regular basis are three times as likely to develop carpal tunnel
syndrome as are adolescents who do not play video games. Therefore, federal
legislation that strictly prohibits the sale of video games to minors would help
curb this painful wrist condition among adolescents.

Premise: Adolescents who play video games on a regular basis are three times
as likely to develop carpal tunnel syndrome as are adolescents who do not play
video games.

The keyword ’as established by’ indicates that the line following it forms the
premise.

Conclusion: Federal legislation that strictly prohibits the sale of video games to
minors would help curb this painful wrist condition among adolescents.

The keyword ’therefore’ indicates that what follows is the conclusion.

Example 2:
Fraud has cost the insurance industry millions of dollars in lost revenue. Thus,
congress will pass a stricter fraud control bill since the insurance industry has
one of the most powerful lobbies.

Premise 1:
Fraud has cost the insurance industry millions of dollars in lost revenue.
Premise 2:
The insurance industry has one of the most powerful lobbies.
The keyword indicator here is ’since’.

Conclusion:
Congress will pass a stricter fraud control bill. The keyword indicator for the
conclusion is ’thus’.
gmat.crackverbal.com
CrackVerbal Critical Reasoning Guide|19

Identifying the assumptions

As we have previously discussed, the assumption is what ties the premises and
the conclusion together. If the assumption were to be false, the argument
would break down entirely. To find assumptions in CR questions, read the
argument, aggressively looking for the links between the premises and conclu-
sion.

Example:

Carl has asked Harry to come over to his house on many occasions last year.
Carl wants to redesign his new studio and Harry is a famed architect. Therefore,
Carl is seeking Harry’s assistance with the design of his studio.

Challenge each link made here:

Are Carl and Harry close friends?


Has Carl already engaged someone else to redesign his studio?
Does Harry take on studio redesign projects?

The answer to each of these questions will throw light upon a different angle of
the problem. For instance, if the two are close friends, then Harry’s visit could
be purely social. If Carl has already engaged someone else for this project, then
he is probably not looking to get Harry’s inputs.

Assumptions will usually contain words already mentioned in the prem-


ise/conclusion of the argument. An assumption that is explicitly stated auto-
matically becomes a premise.

gmat.crackverbal.com
CrackVerbal Critical Reasoning Guide|20

Let’s see if you are able to understand the structure of the following argu-
ments.

1. Half of the subjects in an experiment –Group A –were made to drink large


quantities of a popular sugary drink. Afterwards, this group showed lower
cognitive ability than did the other half of the subjects – Group B - did not
consume the sugary drink. Therefore, intake of sugar needs to be controlled
substantially to prevent reduced cognitive functioning.

One sentence summary:

What are the links that can be attacked?

Author’s assumptions:

2. Even though most employers keep the rights to an employee’s inventions,


the employees retain the rights to the books and articles they create. Therefore,
employees should also retain the rights to the computer programs they create,
if any.
One sentence summary:

What are the links that can be attacked?

Author’s assumptions:

gmat.crackverbal.com
CrackVerbal Critical Reasoning Guide|21

3. A study shows that subscriptions to cable television will decline substantially


in the future, because new video streaming services let customers see the
programs that they want to see at their own convenience. Sun Media Enterpris-
es, a cable-television company, therefore, plans to revamp its product line by
eliminating all its cable-television subscription plans and converting its prod-
uct line to focus on online-streaming services instead.
One sentence summary:

What are the links that can be attacked?

Author’s assumptions:

4. More rigorous selection standards have not been the primary cause of the
present decline in the number of candidates applying to private schools. One
has to consider the fact that most students cannot afford the prohibitively high
tuition fees that private institutions charge. Also it has been proven that stu-
dents in private schools do not perform any better academically than students
in public schools.
One sentence summary:

What are the links that can be attacked?

Author’s assumptions:

gmat.crackverbal.com
CrackVerbal Critical Reasoning Guide|22

5. Teaching universities must also aggressively support research. After all, a


significant percentage of their students are undergraduates, and such institu-
tions are clearly obligated to provide them a quality education.
One sentence summary:

What are the links that can be attacked?

Author’s assumptions:

6. Groundbreaking new research shows that a strong emotional attachment


between a mother and her baby may help prevent diseases, boost immunity,
and enhance a child’s IQ. Therefore, mothers must cuddle their babies and
demonstrate love and affection to improve the baby’s growth and develop-
ment.
One sentence summary:

What are the links that can be attacked?

Author’s assumptions:

gmat.crackverbal.com
CrackVerbal Critical Reasoning Guide|23

Answers

1. Summary: Sugar causes reduced cognitive function because Group A showed


lower cognitive ability after drinking the sugary drink.

What are the links that can be attacked in this argument?

The sugary drink reduced cognitive ability.

Explanation: Is it the sugar in the drink that’s causing the reduction in cognitive
functioning and nothing else? Perhaps there are some additives in the drink,
that are the actual cause? What if Group A already had lower cognitive ability
than that of Group B?

Assumptions: There is no other cause for reduced cognitive functioning in


Group A apart from sugar.

2. Summary: Employees retain rights of books/articles. Therefore, they should


also retain the rights for computer programs.

What are the links that can be attacked in this argument?

Books are the same as computer programs OR computer programs are not
inventions.

Explanation: The assumption that computer programs are not inventions but
belong to the same category as books can be attacked.

Assumption: Books = computer programs

gmat.crackverbal.com
CrackVerbal Critical Reasoning Guide|24

3. Summary: Convert from cable TV to online stream because it is good for the
company
What are the links that can be attacked in this argument?

• Converting will result in profitability

• Potential side effects (harm/benefit)

Explanation: The cost of conversion could potentially outweigh any increased


revenue that converting to online streaming may provide.

Assumption: The main assumption made here is that video streaming is the
only product line to focus on.

4. Summary: Rigorous selection standards are NOT the cause of decline in


applications to private schools

What are the links that can be attacked in this argument?

Expense is a cause for decline in the number of candidates.

Explanation: Perhaps students who typically apply to private schools don’t care
about high tuition fees. The term “most students” is loose and need not include
such students. The performance of students need not necessarily be a reflec-
tion of the teaching or pedagogy at private schools. Perhaps factors other
than academics could be drivers for students applying to private schools. These
possibilities are not considered.

Assumption: Students who apply to private schools are concerned about high
tuition fees.

gmat.crackverbal.com
CrackVerbal Critical Reasoning Guide|25

5. Summary: Teaching universities for undergraduate students must also


support research to provide students with quality education.

What are the links that can be attacked in this argument?

The definition of “quality education”

Explanation: Can’t quality education be provided through teaching and other


activities?

Is research necessary for education to attain a certain quality?

Assumption: Research in universities impacts the quality of undergraduate


education positively.

6. Summary: Mothers must cuddle their babies to show love and affection to
improve the baby’s growth and development.

What are the links that can be attacked in this argument?

Link between demonstration of affection and strong emotional attachment.

Explanation: Is cuddling and demonstrating affection the only way to build a


strong emotional attachment between a mother and her baby? Perhaps there
are less demonstrative mothers who have strong emotional bonds with their
babies. Does prevention of diseases, boost in immunity and enhancement of IQ
necessarily result in improved growth of babies?

Biggest assumption: Cuddling and demonstrating affection indicates a strong


emotional attachment between a mother and her baby.

gmat.crackverbal.com
CrackVerbal Critical Reasoning Guide|26

You will come across 5 types of assumption-based questions on the GMAT.

1. Find the assumption

2. Strengthen the argument

3. Weaken the argument

4. Evaluate the argument

5. Identify the flaw in the argument

Let’s start by looking at the assumption questions.

FIND THE ASSUMPTION QUESTIONS

The first form of assumption-based questions is the fairly straightforward ’Find


the Assumption’ type questions. Such questions will give you an argument and
ask you to identify the underlying assumption. They will use one of these
phrases:

• Assumption/assumed/assumes
• Presupposition/presupposed/presupposes
• Justify the conclusion
• Inserted as an additional premise
• The conclusion cannot be true unless which of the following is true
• The conclusion will be more properly drawn if…

gmat.crackverbal.com
CrackVerbal Critical Reasoning Guide|27

While tackling such questions, it would be useful to look out for choices that
satisfy the following criteria:

1. The assumption will be closely tied to the conclusion


2. It will support/strengthen the conclusion
3. It is very likely to contain words found in the premise and the argument itself
4. Upon negation of the right answer (the assumption), the argument will break

THE NEGATION TECHNIQUE

An argument is true only if its underlying assumption is true, i.e., if the assump-
tion is broken, the argument is broken. When you are down to the last 2 answer
choices, negate each of them. The one that breaks the argument upon negation
is the correct assumption.

Illustrative example

In order to cut costs, some of Company P’s manufacturing plants converted


from oil fuel to LPG last year, when the cost of oil was more than the cost of
LPG. Because of a sudden, unanticipated shortage, however, LPG now costs
more than oil, the price of which has fallen steeply over the past year. The cost
of conversion back to oil would more than negate any cost savings in fuel. So
Company P’s fuel costs this year will be significantly higher than they were last
year.

Which of the following is an assumption on which the argument above de-


pends?
(A)The increase in the cost of fuel cannot be offset by reductions in other
operating expenses.
(B)Company P does not have money set aside for the increased costs of fuel.
(C)The price of LPG will never again fall below that of oil.
(D)The cost of fuel needed by those of Company P’s plants that converted to
LPG is not less than the cost of fuel needed by those plants still using oil.
(E)The price of oil will not experience a sudden and steep increase.

gmat.crackverbal.com
CrackVerbal Critical Reasoning Guide|28

What do we understand from this argument?

Last year: Costoil>CostLPG; some plants moved from oil to LPG

This year: CostLPG>Costoil

The conclusion is that company P’s fuel costs this year will be significantly
higher than last year.

Option A: Whether Company P has the money to meet this cost or not is irrele-
vant.

Option B: Whether the increased fuel cost can be offset or not is again, irrele-
vant.

Option C: First of all, ’will never’ is out of scope. What if the prices fall only 3 or
5 years from now? The question is all about the company’s costs this year. So
this choice is also incorrect.

Option D: Let’s negate this: the cost of the LPG plants is less than that of the oil
plants. In that case, their running costs would be lower than that of the
oil-based plants. This hurts the argument that company P’s costs will rise
significantly this year. Thus, this is the correct answer.

Option E: Let’s negate this: the price of oil will experience a sudden, sharp rise.
This will only increase the costs of the company.

gmat.crackverbal.com
CrackVerbal Critical Reasoning Guide|29

CRACKVERBAL’S 5-STEP APPROACH TO GMAT CR

Step One: Read and Identify

Read the argument carefully and identify the question type and the core struc-
tural pattern of the argument.

Step Two: Analyze

Try to summarize the main point of the argument in a single line in your own
words. Look for keywords that help you identify the premise, conclusion,
counter-conclusion etc.

Step Three: Pre-Phrase

Try to formulate the possible answer on your own, without looking at the
choices.

Step Four: Eliminate

Eliminate the “so what?” answer choices that are irrelevant to the argument.
Keep the ones that you are unable to eliminate, or which seem likely answers.

Step Five: Resolve

Compare the remaining choices for subtle differences and apply the suggested
technique based on question type to arrive at the correct answer.

gmat.crackverbal.com
CrackVerbal Critical Reasoning Guide|30

gmat.crackverbal.com
CrackVerbal Critical Reasoning Guide|31

3. Five common traps in Assumption based Questions

• Causality Versus Correlation


• Comparison
• Statistical
• Harm/Benefit
• Language Shift
• Practice Questions: Find the Assumption

gmat.crackverbal.com
CrackVerbal Critical Reasoning Guide|32

gmat.crackverbal.com
CrackVerbal Critical Reasoning Guide|33

Many GMAT questions have arguments based on flawed logic and you
need to be able to identify the logical errors in these to get the right answer,
even if the question does not ask you to spot the flaw directly.

The good news, however, is that there are only a limited number of such
logical flaw concepts that you need to be familiar with. Let us look at them one
by one:

CAUSALITY VERSUS CORRELATION

Many assumption-based questions test you on causation, a concept you need to


be familiar with. Let us look at a situation where X is causing Y.
Example:
The bell goes off (Y) whenever the teacher comes into the class (X).
But before making an assumption that the teacher’s entry into the class causes
the bell to ring, you need to consider the following:

3 possible scenarios could be in action here:

(a) Z à Y
Here, Z is an alternate cause of Y. One of the answer choices may actually
contain Z. Don’t jump in and eliminate Z as being out of scope. In the above
example, Z could be that the peon rings the bell everyday at the same time as
the class begins, and this is the same time that the teacher also enters the class.

(b) Y à X
Y is causing X and not the other way round. Ensure that reverse causation did
not happen. In the example, the teacher could be entering the class upon
hearing the bell ring and not vice versa. This is called reverse causation.

gmat.crackverbal.com
CrackVerbal Critical Reasoning Guide|34

(c) Zà X and Y
Ensure that there is a cause-effect relationship between X and Y and not just
correlation, i.e., some other cause, Z, may be the reason for both X and Y, and
there could be absolutely no connection between X and Y. For instance, at
exactly 1pm both the teacher enters the class and the bel automatically goes
off. Therefore, those two happen simultaneously but do not have a cause -
effect relationship.

Behavior/qualities of cause-effect relationships

(a) If X happens then Y will definitely happen.


If it rains, the streets will get water-logged.
This is a straightforward cause-effect relationship.

(b) If X doesn’t happen then Y may or may not happen


If it doesn’t rain, the streets may or may not get water-logged.
Rain needn’t be the only reason for the streets getting water-logged. For in-
stance, if a water pipe breaks, but it doesn’t rain, the streets could still get
water-logged.

(c) If Y happens then X may or may not happen


People who eat a lot of junk food are often overweight.
But just because someone is overweight doesn’t necessarily mean that he/she
eats a lot of junk food. Their weight could be because of a lack of exercise or
even a medical or genetic condition.

(d) If Y doesn’t happen then definitely X has not happened


The roads are not wet. So it cannot have rained.
This is the direct opposite of what we saw in the first example. If it rains, the
roads must get wet. If the roads aren’t wet, then it cannot have rained.

gmat.crackverbal.com
CrackVerbal Critical Reasoning Guide|35

Before you jump in and accept the causal relationship at face value, you
need to think of the underlying assumptions.

A typical erroneous assumption here is that there is indeed a cause-effect


relationship existent between events X and Y. But as we previously saw, this
need not necessarily be true.

X and Y could be happening simultaneously (have a correlation) but it is


not necessary that one causes the other.

So how would you weaken such an argument?

You can do so by introducing an alternate cause or reason for the event.

The opposite is also true: to strengthen such an argument, we should remove


any alternate cause or reason for the event.

EXAMPLE

Researchers have noticed that people who have very low levels of potassium in
their blood develop Alzheimer’s disease in the later stages of their adult lives.
Therefore, the researches recommend that people include potassium rich foods
such as greens, beans, sweet potatoes and bananas in their daily diet to prevent
the onset of Alzheimer’s disease.
How would you question this conclusion?

Premise: The argument says that the people who were observed to have very
low levels of potassium also developed Alzheimer’s disease in the later stages
of their lives.
Conclusion: People should eat more potassium rich food to prevent the onset
of Alzheimer’s disease.

gmat.crackverbal.com
CrackVerbal Critical Reasoning Guide|36

In essence the argument says that X --> Y (X causes Y)

where

X = low levels of potassium in blood

Y = onset of Alzheimer’s disease

But what if the onset of Alzheimer’s has nothing to do with the potassium
levels? Perhaps Alzheimer’s is an inherited disease and one of the other effects
of the Alzheimer’s gene is suppressed blood potassium level?
Therefore let’s assume this possible cause as Z
Z = a gene that causes Alzheimer’s
If it were to be found that the people observed by the researchers had this
gene (Z), it would be very likely that Z --> Y (Z caused Y)

So, whenever you see an argument in GMAT CR with a seemingly causative


relationship, make sure that you look for an alternative cause.

COMPARISON

In some arguments, the conclusion is based on similarity between two things.

The typical erroneous assumption in these is often that ’similar’ means ’the
same’.

Such arguments can be weakened by showing that the things compared are
different in some way(s) and can be strengthened by eliminating a possible
difference between them.
Example:

Greenville, like Cityville, is growing as a metropolis, partly due to the influx of


immigrants from other states. Hence, just as it happened in Cityville in the
1950s and 1960s, public hygiene will be Greenville’s biggest challenge in the
years to come.
gmat.crackverbal.com
CrackVerbal Critical Reasoning Guide|37

What is the underlying assumption?

The assumption is that Greenville’s public services are the same as Cityville’s.
But what if Greenville’s government has foreseen this danger and has made
appropriate provisions to address the hygiene question? Or say Greenville has
a much smaller area than Cityville; in that case, housing and traffic could be its
biggest challenges, overriding public hygiene.

Thus, by showing that these two cities are dissimilar in different ways, we can
weaken the argument.

STATISTICAL TRAPS

In such questions, the argument is based on the results of a sample, poll or


survey.

Example :

People in urban areas are eating more junk food (classified as burgers, cola and
pizza) these days because from 2001 to 2009, the average house-hold spend-
ing on such junk food in urban areas has increased from 5% to 12%.

What is the problem with this reasoning?

Answer: Here, we are assuming that just because the spending on junk food has
increase, it means that the consumption of junk food has also increased. This
need not necessarily be true. For instance, the cost of such junk food could
have dramatically increased from 2001 to 2009, while the consumption re-
mained the same (or even decreased) and hence, the spending could have gone
up.

gmat.crackverbal.com
CrackVerbal Critical Reasoning Guide|38

Example :

Examination of the Allies’ fighter planes that survived during WW II showed


that the front portions of the crafts were undamaged. Therefore, defence
analysts concluded that the back portions of the crafts were weaker and re-
quired redesign.

Answer: The problem is that the sample we are considering is the planes that
survived. But what about all the planes that did not survive? They could have
been damaged in the front or the back portions! Then our conclusion that the
back portion of the aircraft is weaker is no longer valid. This is a fine example of
selection bias leading us astray.

Tackling statistics-based / numerical CR questions on the GMAT

Often, the GMAT test-setter complicates things by citing lots of numerical data
such as proportions, percentages, rates, etc. However, at the crux, numerical CR
questions usually involve just 3 ideas:

1. A total figure
2. A number within this total
3. A percentage of this total

The most common errors made on such types of questions are:

· Assuming that an increase/decrease in numbers automatically lead to an


increase/decrease in percentages

· Assuming that an increase/decrease in percentages automatically lead


to an increase /decrease in numbers

gmat.crackverbal.com
CrackVerbal Critical Reasoning Guide|39

Example :

Citrix Computers’ market share has increased from 14% in 2010 to 27% in
2012. This means that they are now selling more computers than they did in
2010.

Sounds plausible?
Unfortunately, this argument is flawed.

What is the market share?

Market share (%) = (Volume sales of the Citrix / Total volume sales of all the
competitors in the industry) * 100. If the market share of Citrix has increased
from 2010 to 2012, it is not necessarily because of an increase in volume sales;
it could also result from a decrease in overall industry sales.
Thus, an increase in percentage figures does not automatically translate to an
increase in absolute numbers. The reverse is also true.

Example:

200 new primary schools have been established in Vargonia by the Ministry of
Education this year. Thus, Vargonia will have a greater share of the total num-
ber of primary schools in the nation. The share of Vargonia in the nation’s
market (%) = No. of primary schools in Vargonia / Total no. of primary schools
in the nation

This year, 200 new primary schools have come up in Vargonia, but what if the
same has happened in other states in the nation? The denominator would have
a much bigger increase compared with the numerator; Vargonia’s share may
remain the same or even decrease. Thus, an increase in absolute numbers also
need not necessarily result in an increase in percentages.

gmat.crackverbal.com
CrackVerbal Critical Reasoning Guide|40

Things to remember while tackling statistics-based CR questions

1. Any time you see that the question stem moves from one type of statistic to
another, pay attention. For instance, total vs. (per X), numbers vs. propor-
tions/percentages, general trends vs. specific changes, etc.

2. A smaller percentage of a larger number can be bigger than a larger per-


centage of a smaller number

3. Be wary of answers that quote absolute numbers when the question talks
only in percentages/proportions, and vice versa.

4. Most importantly, not all CR questions which quote statistics actually require
you to work with them —in many cases, the statistics really do not matter!

HARM/BENEFIT

Some arguments recommend a course of action to avoid harm or gain some


benefit. The typical erroneous assumption is that no equal or greater benefit or
harm respectively can come from the action deemed harmful or beneficial.
Thus, by showing a potentially greater benefit/harm, we can weaken such
arguments, and by removing them, we can strengthen such arguments.
Example:

A new wonder drug has been discovered which has been proven to improve the
heart’s functioning by more than 150%. Hence, the new drug should be rec-
ommended by doctors to patients suffering from heart diseases.

Here, we have not considered the possible side effects of the wonder drug.
What if it improves heart efficiency, but at the same time, increases the risk of
cancer?

By showing that an equal/greater harm can come from taking this drug, we are
weakening the argument that it should be prescribed to heart patients.

gmat.crackverbal.com
CrackVerbal Critical Reasoning Guide|41

LANGUAGE SHIFT

Certain arguments use different words in the premises and conclusion to


describe a common thing, situation, or event, thus confusing the test-taker. The
typical erroneous assumption in such cases is that the different words or
phrases mean the same thing or encompass the same idea.

By identifying that the shift in language results in inaccurate description or


logic, we can weaken this argument. Similarly, by removing the discrepancy or
ambiguity between terms or ideas, we can strengthen the argument.

Example:

These fans are certainly best-in-class because they are tested by the highest
fan certifying authority in the world —International Institute of Fans.

The conclusion here talks about the fans being the ’best in class’ but the only
thing we know about the fans is that they were ’tested’ by the fan certifying
authority. The flaw in this argument, therefore, is the ambiguity that exists
between the terms ’tested’ and ’best in class’. We need to address this ambigu-
ity (find out if testing equals best) to understand whether the conclusion drawn
here is valid.

IDENTIFYING STRUCTURAL PATTERNS IN GMAT CR

A key to doing well in GMAT CR (especially assumption-based questions) is the


ability to identify structural patterns and correlate them while solving other
questions.

Once you understand an argument, think of it structurally and not in terms of X


and Y, and map it onto a real life examples that is easy for your brain to process.
You will find that the logic will “click”!

gmat.crackverbal.com
CrackVerbal Critical Reasoning Guide|42

Example:

Sviatovin is a medieval text whose author and exact date of composition are
unknown. However, the events in the life of Prince Sviatov that the text de-
scribes occurred in 1165, and in the diagram of Sviatov’s family that accompa-
nies the text, his father (who died in 1167) is identified as still living. Thus,
Sviatovin must have been written between 1165 and 1167.

Understand this argument and try to correlate it with a real life everyday exam-
ple.

I don’t know when my great grandmother died. But I have a letter she wrote to
my grandfather, dated 1897. This means that she was alive in 1897.

Now that you have a parallel argument in mind, it is much easier to cut through
the complex language and the clutter of facts and get to the crux of the argu-
ment!

gmat.crackverbal.com
CrackVerbal Critical Reasoning Guide|43

PRACTICE QUESTIONS: FIND THE ASSUMPTION

1. Recent research indicates that mercury levels in the bodies of saltwater fish
are higher now than they were a hundred years ago. Mercury from the fish
accumulates in the base of the feathers of seabirds that eat saltwater fish.
Feathers taken from seabirds stuffed and preserved in the 1900s were found to
contain only half as much mercury as do feathers recently taken from living
birds of the same species.

The argument depends on the assumption that

(A) The number of sources of pollutants in 1990s was much lower than it is
now.
(B) The quantity of mercury found in the body of a saltwater fish depends on
the amount of pollution in the ocean habitat of the fish.
(C) The same techniques used today were used to stuff and preserve birds in
the 1900s.
(D) The process used to preserve birds in the 1900s did not substantially de-
crease the amount of mercury in the birds’ feathers.
(E) The proportion of saltwater fish in the diet of seabirds has remained the
same since the 1900s.

gmat.crackverbal.com
CrackVerbal Critical Reasoning Guide|44

2. Modern physicians often employ laboratory tests, in addition to physical


examinations, in order to diagnose diseases accurately. Insurance company
regulations that deny coverage for certain laboratory tests therefore decrease
the quality of medical care provided to patients.

Which one of the following is an assumption that would serve to justify the
conclusion above?

(A)Laboratory tests are more costly to perform than are physical examinations.
(B)Many physicians generally oppose insurance company regulations that, in
order to reduce costs, limit the use of laboratory tests.
(C) No disease can be accurately diagnosed through physical examination
alone.
(D)Many patients who might benefit from the uncovered laboratory tests do not
have any form of health insurance.
(E) Physical examinations and the uncovered laboratory tests together provide
a more accurate diagnosis of many diseases than do physical examinations
alone.

gmat.crackverbal.com
CrackVerbal Critical Reasoning Guide|45

3. Economic considerations color every aspect of international dealings, and


nations are just like individuals in that the lender sets the terms of its dealings
with the borrower. That is why a nation that owes money to another cannot
ever become a world leader.

The reasoning in the passage assumes which one of the following?

(A) A nation that does not lend to any other nation cannot be a world leader.
(B)A nation that can set the terms of its dealings with other nations is certain to
be a world leader.
(C)A nation that has the terms of its dealings with another set by that nation
cannot be a world leader.
(D)A nation that is a world leader can borrow from another nation as long as
that other nation does not set the terms of the dealings between the two
nations.
(E)A nation that has no dealings with any other nation cannot be a world leader.

gmat.crackverbal.com
CrackVerbal Critical Reasoning Guide|46

4 .Despite improvements in treatment for bronchitis, the death rate from this
disease has doubled during the past decade. Two possible explanations for this
increase have been offered. First, the recording of deaths due to bronchitis has
become more widespread and accurate in the past decade than it had been
previously. Second, there has been an increase in urban pollution. However,
since the rate of deaths due to bronchitis has increased dramatically even in
cities with long-standing, comprehensive medical records and with little or no
urban pollution, one must conclude that the increased death rate is due to the
use of nebulizers by bronchitis sufferers.

Which one of the following is an assumption on which the argument depends?

(A) Urban pollution has not doubled in the past decade.


(B)Increased urban pollution, improved recording of bronchitis deaths, and the
use of bronchial nebulizers are the only possible explanations of the increased
death rate due to bronchitis.
(C) Bronchial nebulizers may cause side effects even when used according to
the recommended instructions.
(D)The use of bronchial nebulizers may aggravate other diseases that fre-
quently occur among bronchitis sufferers.
(E) The death rate from bronchitis has shown a similar increase even in cities
with heavy urban pollution.

gmat.crackverbal.com
CrackVerbal Critical Reasoning Guide|47

5. Nitrobacter, a bacteria living in the roots of bean plants or other legumes,


produces fixed nitrogen, an essential plant nutrient that, for non-legume crops
such as wheat, must be supplied by applications of nitrogen-based fertilizer. So
if biotechnologists succeed in producing wheat strains whose roots can play
host to nitrobacter, the need for nitrogen-based fertilizers will be reduced.

The argument above makes which one of the following assumptions?

(A) Fixed nitrogen produced by nitrobacter is beneficial to wheat and can


replace artificial fertilizers
(B) Nitrobacter living in the roots of wheat would be able to produce fixed
nitrogen.
(C)Fixed nitrogen is currently the only soil nutrient for wheat that must be
supplied by artificial fertilizers.
(D) There are no naturally occurring strains of non-legume plants that have
nitrobacter living in their roots.
(E)Legumes are currently the only crops that produce their own supply of fixed
nitrogen.

gmat.crackverbal.com
CrackVerbal Critical Reasoning Guide|48

6. The 15th century polymath Leonardo Da Vinci is renowned for his work in art,
engineering and biology. But Da Vinci also conducted experiments secretly for
many years based on the arcane theories of alchemy, a precursor to the rela-
tively modern science of chemistry, trying unsuccessfully to transmute com-
mon metals into gold and produce rejuvenating elixirs. If more alchemists
bygone centuries had published the results of their experiments, the
knowledge of chemistry in subsequent centuries could have been much more
advanced.

Which one of the following assumptions would allow the conclusion concerning
chemistry to be properly drawn?

(A) Scientific progress is retarded by the reluctance of scientists to publish the


results of their unsuccessful experiments.
(B) Da Vinci’s work in alchemy, like his works in art, engineering, and biology,
would also have achieved renown if it had been published.
(C) No alchemist in the 15th century, including Da Vinci, ever published the
results of their experiments.
(D) Advances in science are hastened when reports of experiments, whether
successful or not, are available for review and study.
(E) The 15th century alchemists could have achieved their goals only if their
experiments had been subjected to public scrutiny.

gmat.crackverbal.com
CrackVerbal Critical Reasoning Guide|49

7. Senator Woods: The government’s funding program for the sciences is


intended to encourage the creation of works of scientific excellence. However,
a government-funded science program can never reflect the scientific con-
science of the scientist because scientists, like anyone else who accepts
financial support, will inevitably try to please those who control the distribu-
tion of that support. Thus, government funding of the sciences is not only a
waste of taxpayers’ money, but it also cannot lead to the creation of works of
true scientific excellence.

Which one of the following is an assumption on which Senator Wood’s argu-


ment is based?

(A)Once a scientist has produced works of true scientific excellence, he or she


will never accept government funding.
(B)A work of science that does not reflect the scientific conscience of the
scientist cannot be a work of true scientific excellence.
(C)Distribution of government funds for the sciences is based on a broad
agreement as to what constitutes scientific excellence.
(D)Many taxpayers are concerned about the purposes for which their tax money
is utilized.
(E)The government bodies that control scientific funding will discourage scien-
tists from abiding by their scientific conscience.

gmat.crackverbal.com
CrackVerbal Critical Reasoning Guide|50

8. Research papers in psychology that are co-authored by psychologists inter-


nationally are cited in subsequent papers about 60% more often than are
those papers without international collaborations. This shows that research
projects conducted by international research teams are more significant than
those conducted by individual researchers.

Which one of the following is an assumption on which the argument depends?

(A) The number of citations a research paper receives is a measure of the


significance of the research.
(B)It is possible to ascertain whether a research paper is a product of interna-
tional collaboration by determining the number of citations it has received.
(C) No research paper by an individual researcher has been cited more times in
subsequent papers than have collaborative papers in the same field.
(D) The collaborative efforts of scientists who are citizens of the same country
do not produce research papers that are as important as papers that are pro-
duced by international collaboration.
(E) Researchers will not inflate the number of citations they receive by citing
themselves in subsequent journals.

gmat.crackverbal.com
CrackVerbal Critical Reasoning Guide|51

9. Although the present supply of crude oil for the purpose of energy produc-
tion is adequate for the current levels of use, the increase in population over
the next few decades will drastically increase demand for energy. Hence, we
must focus on the development of alternate fuel sources such as wind power
and bio-thermal energy to meet the energy demands in the near future.

Which one of the following is an assumption required by the argument?

(A) Wind and bio thermal energy are the only sources through which the energy
requirements of the world can be met in coming decades.
(B) The total supply of crude oil will diminish in coming decades.
(C) The supply of crude oil will not increase sufficiently to meet the increasing
energy demand in coming decades.
(D) If the population were to not increase drastically in coming decades, the
supply of crude oil would be sufficient for energy production.
(E) No measures to develop alternate sources of energy have been attempted
so far.

gmat.crackverbal.com
CrackVerbal Critical Reasoning Guide|52

10. Recent studies have shown that pilot error contributes to two-thirds of all
crashes in commercial airlines. To address this problem, the airline industry has
upgraded its training programs by increasing the hours of classroom instruction
and emphasizing communication skills in the cockpit. Yet, it is unrealistic to
expect such measures to compensate for pilots’ lack of actual flying time.
Therefore, the airline industry should rethink its training approach to reduce
commercial flight crashes.

Which one of the following is an assumption upon which the argument de-
pends?

(A) Well-designed training programs can eliminate commercial flight crashes.


(B) Classroom instruction and communication skills training are not relevant for
commercial flight pilots.
(C) The number of airline crashes will decrease if pilot training programs focus
on increasing actual flying time.
(D) Lack of actual flying time is an important contributor to pilot error in com-
mercial plane crashes.
(E)Recent studies about the causes of commercial airline crashes are accurate.

gmat.crackverbal.com
CrackVerbal Critical Reasoning Guide|53

ANSWER EXPLANATIONS

1. Answer: D
The argument states that mercury levels in fish are now higher. This is based on
the evaluation of the amount of mercury in bird feathers. The assumption here
is that analysis of the stuffed birds’ feather is reflective of the actual level of
mercury at that time. Option D addresses this. This bridges the gap and bolsters
the argument.

(A) The number of pollution sources is irrelevant to us.


(B)This is a broad statement unsupported by the argument. Negation does not
break the argument.
(C) It is not necessary that the techniques used are the same, as long as the
preservation is intact. Negation does not break the argument.
(D) On negation, this answer choice breaks the argument. “The process used to
preserve birds in the 1900s did substantially decrease the amount of mercury
in the birds’ feathers.” If the above is true, then probably the mercury levels are
the same as before or were even higher in the past. Hence, this is the correct
answer.
(E) We don’t know, can’t say that the proportion of fish in the diet of the birds
impacts the quantity of mercury found in the birds’ feathers.

gmat.crackverbal.com
CrackVerbal Critical Reasoning Guide|54

2. Answer: E

The assumption made is that denying certain laboratory tests may lead to
inaccurate diagnosis of diseases. Option E is the most befitting in this context.

(A) The cost of the lab tests is irrelevant.


(B) Whether physicians oppose the insurance company regulations is irrelevant.
(C) This is an extreme answer choice. Let’s negate it: some diseases can be
accurately diagnosed through physical examination alone. But what about
other diseases? This does not break the argument.
(D)This is talking about an irrelevant sample —patients with no health insur-
ance at all. The argument’s concern is those who have insurance but are denied
coverage for certain tests.
(E) On negation, this answer choice breaks the argument: “Physical examination
and the uncovered laboratory tests together do not provide a more accurate
diagnosis that...” If this statement is true, the insurance company regulations do
not affect the quality of medical service provided. Hence, this is the correct
answer.

gmat.crackverbal.com
CrackVerbal Critical Reasoning Guide|55

3. Answer: C

The argument states that when a country owes money to another, it cannot
become a world leader. The only option that is relevant to the conclusion and
corroborates with the premise is option C, which states that when another
country sets terms the country bound by those terms cannot become a world
power.

Let the nations be B(orrower) and L(ender). The argument says that L always
sets the terms of its dealings with B. Therefore, B can never be a world leader.

(A) It says that being a lender is necessary to be a world leader. This is incorrect.
(B) It says that being a lender is a guarantee that the nation would be a world
leader. This is incorrect.
(C) On negation, this answer choice breaks the argument: “A nation that has the
terms of its dealings with another set by that nation can be a world leader.” If
this sentence holds true, B can become a world leader. Therefore, this is the
correct answer.
(D) It goes against the premise. It says that B can be a world leader as long as
the lender does not set the terms of the deal. But the premise is that the lender
always sets the terms of the deal.
(E)We cannot comment on this as the argument mentions nothing to this effect.
Therefore, we don’t know; can’t say (DKCS).

gmat.crackverbal.com
CrackVerbal Critical Reasoning Guide|56

4. Answer: B

The argument follows a causation-correlation flaw pattern. The assumption


here, therefore, is that there are no other factors in play for the cause of bron-
chitis deaths. Option B is the only option that addresses this. This is therefore
the apt answer choice.

(A) It doesn’t matter by how much urban pollution has increased, because the
death rate is higher even in cities where urban pollution is hardly present.
(B) On negation, this answer choice breaks the argument: “Increased urban
pollution, improved recording of bronchitis deaths, and the use of bronchial
nebulizers are not the only possible explanations of the increased death rate
due to bronchitis.” If this sentence holds true, the use of nebulizer need not be
the only reason for death due to bronchitis. Therefore, this is the correct an-
swer.
(C)It equates “side effects” to “death” —the side effects could be very minor.
For instance, rashes or spots need not lead to death.
(D) Again, other diseases may be aggravated, but these may not be serious and
need not necessarily lead to death.
(E) Negation does not break the argument. It does not provide the missing
connect between bronchial nebulizers and death rate.

gmat.crackverbal.com
CrackVerbal Critical Reasoning Guide|57

5. Answer: B

The biggest assumption made in this argument is that this plan will have the
desired effect. For the plan to work, the nitrobacter in the genetically engi-
neered strains of wheat must actually be able to produce fixed nitrogen.

(A)This choice is incorrect because the argument is talking about reducing the
need for nitrogen -base fertilizers —not replacing any generic fertilizer.
(B)Let’s negate this answer choice: “Nitrobacter living in the roots of wheat
would not be able to produce fixed nitrogen.” If the above sentence holds true,
there won’t be any point in producing wheat strains whose roots play host to
nitrobacter. Therefore, this is the correct answer.
(C) It doesn’t matter whether fixed nitrogen is the only nutrient that must be
supplied artificially
(D) It doesn’t matter whether there are naturally occurring strains. We are
concerned with the development of specific wheat strains.
(E) It doesn’t matter whether legumes are the only crops.

gmat.crackverbal.com
CrackVerbal Critical Reasoning Guide|58

6. Answer: D

The argument states that chemistry would have become more advanced if
alchemists published their results. The assumption here is that doing this
would lead to the desired consequence, or in other words - publishing results
of experiments irrespective of how successful they are would result in ad-
vancement in chemistry. Option D is the closest to this, and is therefore the
correct answer.

(A) This is a broad generalization. The argument is specific to alchemy and


chemistry, not scientific progress in general.
(B) This is a prediction not necessarily supported by the argument. This option
does not provide the missing link between alchemy in the 15th century and
chemistry in subsequent centuries. Negation doesn’t break the argument.
(C) This goes against the premise. The premise says “If more alchemists had…”,
indicating that a few could have published their experiments. Negation does
not break the argument.
(D) On negation, this answer choice breaks the argument: “Advances in science
are not hastened when reports of experiments, whether successful or not, are
available for review and study.” If this sentence holds true, the knowledge of
chemistry in subsequent years could not have been more advanced. Therefore,
this is the correct answer.
(E) Again, this is not necessarily supported by the argument.

gmat.crackverbal.com
CrackVerbal Critical Reasoning Guide|59

7. Answer: B

The argument states that if funded by the government, “true scientific excel-
lence” cannot happen. This is stated because funding reduces the amount of
scientific conscience reflected in the work of the scientist. Therefore, the
author assumes that scientific conscience is required to produce a work of true
scientific excellence. Option B states this and is the correct response.

(A)This is not supported by the argument.


(B) On negation, this answer choice breaks the argument: “A work of science
that does not reflect the scientific conscience of the scientist can be the work
of true scientific excellence.” If the above sentence holds true, works of true
excellence can be created, and funding is not wasted. Therefore, this is the
correct answer.
(C) This is a vague answer choice that does not specify the terms of the agree-
ment. Negation does not break the argument.
(D) This is a broad statement —how many is “many”? Secondly, it does not
matter whether taxpayers are concerned about how their tax money is utilized
—even if they are not concerned, it could still be a waste of tax money.
(E) This is an extreme answer choice. Even if the government bodies do not
discourage scientists from abiding by their conscience, the scientists may
themselves feel obliged to the financial supporter.

gmat.crackverbal.com
CrackVerbal Critical Reasoning Guide|60

8. Answer: A

The link here is between being “more significant” and being “cited more.” The
assumption therefore is that being cited more is a measure of a paper’s signifi-
cance.

(A) Let’s negate this answer choice: “The number of citations a research paper
receives is not a measure of the significance of the research.” If this sentence is
true, research projects conducted by international teams won’t be more signif-
icant than those by individual researchers. Therefore, this is the correct answer.
(B) No such exact figures can be arrived at based on the information in the
argument. It is irrelevant to the core conclusion.
(C) Let’s negate this: some research papers by individual researchers have been
cited more times than have collaborative papers in the same field. However,
this is an isolated statistic and doesn’t necessarily go against the main statistic
quoted. We don’t know this number. So, this does not break the argument.
(D)We cannot comment on the significance of collaborative research by scien-
tists from the same country. The supporting information in the argument is
insufficient.
(E) Let’s negate this: researchers will inflate the number of citations they re-
ceive. But who inflates more —individuals or collaborators? And by how much?
Without these numbers, this option does not break the argument.

gmat.crackverbal.com
CrackVerbal Critical Reasoning Guide|61

9. Answer: C

The argument states that alternate fuel sources must be the focus in coming
decades. The assumption is that crude oil sources will not be sufficient to
maintain the growing demand. Option C is a rephrasing of this.

(A) There could be other alternative energy sources —the argument mentions
these as examples
(B) This is an extreme answer choice. Let’s negate this: the total supply of crude
oil will not diminish in coming decades. It could either remain the same or
increase. But with the rise in population, even if the supply remains steady, an
energy crisis could occur. Negation does not break the argument.
(C) Let’s negate this answer choice: “The supply of crude oil will increase
sufficiently to meet the increasing energy demand in the near future.” If this
sentence holds true, there won’t be any need to develop alternate fuel. There-
fore, this is the correct answer.
(D) We don’t know for sure that the population is the only reason for the antic-
ipated energy crisis. It is a reason —but it could be one of many reasons.
(E) The argument merely says that the focus henceforth must be on alternative
sources. We don’t know, can’t say that the focus has not been there previously.

gmat.crackverbal.com
CrackVerbal Critical Reasoning Guide|62

10. Answer: D

The argument suggests that actual flying time be increased to reduce commer-
cial flight crashes. The reasoning behind this is that the author assumes that
such pilot errors are caused by lack of actual flying time. Option D summarizes
this logic.

(A) Training programs may impact the pilots’ readiness but are no guarantee
that commercial flight crashes can be eliminated.
(B) This is an extreme answer choice not supported by the argument. The
argument only says that these cannot compensate for actually flying time —not
that these are useless.
(C) This talks about airline crashes in general and not commercial airline crash-
es.
(D) Let’s negate this answer choice: “Lack of actual flying time is not an im-
portant contributor to pilot error in commercial plane crashes.” If this holds
true, the airline industry need not rethink its training approach. Therefore, this
is the correct answer.
(E) This is a vague answer choice. We don’t know whether this option is refer-
ring to the same studies quoted in the argument. It also refers broadly to
“causes of commercial airline crashes, whereas the study in the argument could
have been something specific.

gmat.crackverbal.com
CrackVerbal Critical Reasoning Guide|63

4. Strengthen and Weaken the conclusion

• Type 2: Strengthen the Conclusion


• Type 3: Weaken the Conclusion
• Practice Questions: Strengthen and Weaken
• Except Questions

gmat.crackverbal.com
CrackVerbal Critical Reasoning Guide|64

gmat.crackverbal.com
CrackVerbal Critical Reasoning Guide|65

TYPE 2: STRENGTHEN THE CONCLUSION

Such questions require the test-taker to identify the option that strengthens
the conclusion. The correct answer choice that does this should fix a potential
weakness in the conclusion by introducing additional supporting evidence.
Upon negation, this choice should weaken the conclusion. Strengthen ques-
tions will use one or more of the following phrases:

• Strengthens
• Supports
• Helps
• Most justifies
• Bolsters
• Buttresses

A strengthen question will always bring information from outside. Any answer
choice in strengthen/weaken should be considered as an additional premise.
Don’t eliminate options assuming they are out of scope.

gmat.crackverbal.com
CrackVerbal Critical Reasoning Guide|66

Illustrative example

Carbon monoxide contains dangerous toxins that cause changes in the respira-
tory system. Because carbon monoxide presents such a high health risk, legisla-
tion is needed to regulate the use of vehicles and lead based fuel.

Which of the following, if true, provides the most support for the argument
above?

(A)The amount of dangerous toxins contained in carbon monoxide is much less


than the amount contained in an equal volume of automobile exhaust.
(B)Within the jurisdiction covered by the proposed legislation, most commuters
use electrically powered vehicles.
(C) Smoke produced by automobile emissions is significantly more toxic than
smoke from stoves used for cooking fuel.
(D) No significant beneficial effect on air quality would result if automobile
emissions were banned within the jurisdiction covered by the proposed legis-
lation.
(E) In valleys where gasoline is used as the primary source of fuel for vehicles,
the concentration of smoke results in poor air quality.

What do you understand from this argument?

The conclusion of the argument is that there is legislation that is needed to


regulate the use of vehicles. The premise given to support this statement is that
carbon monoxide contains dangerous toxins that can be harmful. Hence the
assumption that links both the conclusion and the premise is that the air quality
of that particular place is affected only by the carbon monoxide from vehicle
emissions.

gmat.crackverbal.com
CrackVerbal Critical Reasoning Guide|67

We need to now find an option that agrees with the above statement.
(A) This option doesn’t really tell us anything about the statement above.
(B)This answer choice talks about the proposed legislation but the argument is
talking about whether such a regulation is even needed or not. Therefore, this
is not the correct answer.
(C) This is a contender. However this option doesn’t push a strong reason for
the need for legislation. We don’t know how toxic the smoke from stoves is.
(D) Again, if there was no significant effect, it means that the legislation is
ineffective.
(E) This option tells you that a legislation is necessary for the control of Carbon
monoxide emissions.

TYPE 3: WEAKEN THE CONCLUSION

Such questions require the test-taker to identify the option that weakens the
conclusion. The correct answer choice that does this should identify a faulty
assumption in the argument by introducing additional detracting evidence.
Upon negation, this choice should strengthen the conclusion.

Such questions will use one or more of the following words/phrases:


• Damage
• Counter
• Absurdity
• Vulnerable
• Flaw
• Questionable
• Fallacious
• Weaken
• Attack
• Undermine
• Refute
• Argue against
• Call into question
• Cast doubt
• Challenge
gmat.crackverbal.com
CrackVerbal Critical Reasoning Guide|68

Illustrative example

Latex that can easily be converted to rubber can at present be obtained only
from the bark of the Para rubber tree, a tree that is quite rare in the wild. It
takes the bark of 3000 trees to make one kilogram of latex. It follows, there-
fore, that continued production of Latex must inevitably lead to the Para tree’s
extinction.

Which of the following, if true, most seriously weakens the argument above?

(A)Latex made from the Para tree is dispensed by chemists appointed by the
government.
(B)Latex made from the Para tree is expensive to produce.
(C) The leaves of the Para tree are used in a number of medical products.
(D) The Para tree can be propagated from cuttings and are grown under cultiva-
tion.
(E) The Para tree generally grows in largely inaccessible places.

What do you understand from the argument?

The conclusion of the argument is the fact that continued production of Latex
will lead to the Para tree’s extinction. The premise given to support this state-
ment is that this can be obtained from the bark of the Para rubber tree, and that
it also takes 3000 trees to make one kilogram of latex.

Hence, the argument assumes that continued use of the Para rubber tree to
make latex will cause its extinction. You will need to figure out an answer that
opposes the above statement. This means that your answer must be saying that
the continued exploitation of the Para rubber tree will not lead to its extinction.

gmat.crackverbal.com
CrackVerbal Critical Reasoning Guide|69

(A)This option does not relate to the argument in any way and should therefore
be eliminated.
(B)So what? It being expensive doesn’t have anything to do with extinction.
(C)This strengthens the answer in some way. If the leaves are used in a lot of
products, it will lead to more extinction.
(D)This option weakens the argument by saying that the Para tree will not
become extinct. Rather the same can be propagated as mentioned in the pas-
sage.
(E)This strengthens the argument and should be eliminated.

OUT OF SCOPE ANSWER CHOICES

One tool that most students employ to eliminate answer choices on GMAT CR,
especially Assumption based questions is the “Out of Scope” reason. By defini-
tion an Out of Scope answer choice is an answer choice that brings in an entity
or an identity that is not present in the argument or that is not discussed within
the scope of the argument.

Nevertheless, it is good to remember that one cannot just go by the definition


of Out of Scope blindly while eliminating the answer choices. It is vital to
understand that an Out of Scope answer choice is something that doesn’t affect
the logic of the argument irrespective of whether it has something that is
discussed in the argument or not.
Let us look at a sample demonstrating the same.

“Will the allocation for HR hiring in FMCG companies increase this year?”

If the above statement is the question and we consider two statements as given
below,

1. FMCG companies have increased their outlook for the current year and will
hire at a higher rate this year.

2. The fortune 500 companies have increased their allocation for IT this year,
most of which will be outsourced overseas.
gmat.crackverbal.com
CrackVerbal Critical Reasoning Guide|70

In the given statements, the first statement discusses about FMCG companies
and the increase in their hiring this year. This shows that their HR hiring alloca-
tion will also increase this year. Hence this statement is well within the scope
of the argument.

However, the second statement talks about the fortune 500 companies and
their IT allocation this year. Logically, this would explain why IT hiring will
increase and not the hiring of the FMCG companies. Hence, this is clearly out of
scope.

Also, on a Strengthen or a Weaken question, you need to be extra careful about


employing Out of Scope reasons to eliminate answer choices. This is because, it
is possible for an answer choice to have something from outside the scope of
the argument affect the argument i.e. strengthen or weaken the logic of the
argument.

Tight Rope walk in CR

This is a very important strategy in CR that you will need to keep in mind when
you are studying for CR.

If you consider all the question types in CR like strengthen, weaken, evaluate,
paradox etc. all of these questions require you to judge a scenario by providing
you with external information. (It is very important to realize that you are
provided with external information). You will need to determine whether this
external information will impact the situation given in the question.

This is a struggle that you are going to have from now till the time you take
your GMAT and even people who have a raw score of 45 or 46 in verbal have
faced the same problem.

The problem here is that on CR “there is a tight rope walk” you will have to
perform. If you end up assuming a lot, all answer choices starts looking correct.
This is one extreme situation.

gmat.crackverbal.com
CrackVerbal Critical Reasoning Guide|71

The other extreme situation is that you end up assuming too little, for e.g. An
OG question talks about ‘Bamboo as a construction material and it says it is
more tensile than steel and stronger than concrete and yet it is not used where
the land prices are very high’.

The right answer option here says bamboo is not used in places where the land
prices are very high since it’s a poor construction material for high rise build-
ings.

When you look at all the answer options in this question, the correct answer
choice may seem bizarre. But if you critically think about it, then the underlying
fundamental rule is that in “Areas where the land prices are high, we typically
create vertical structures”.

Now this piece of information is logical and is something that everyone knows.
We will need to bring to the table some amount of common sense knowledge.

Another question similar to the one above speaks about a particular fish and
the catch of this particular fish going down, but despite this, the prices re-
mained constant. Why?

Essentially the right answer choice for this question says that people started
substituting this particular fish with some other fish. In a nutshell, the underly-
ing concept that the GMAT expects you to know is the Elasticity of supply and
demand (if demand remains constant and supply decreases then price increas-
es).

Now one way that the prices also didn’t go up was because the demand went
down. This is where the substitution angle came in. These are all commonsen-
sical thing that you can bring in to the question.

So the two questions that you will be itching to ask is how much we should
assume and how much we should not? Well to sound almost like a Shakespear-
ean, ‘that is the question’
gmat.crackverbal.com
CrackVerbal Critical Reasoning Guide|72

It is definitely going to be a struggle, but as you solve more and more official
questions your judgement will get better and better. But will it ever reach a
point, where for any question you would be able to say ”how much you could
assume” and ”how much you should not”, probably not. But you will get better
at it.

Remember not to be hasty and remove your out of scope gun and start elimi-
nating the options based on words not mentioned in the question. Keep in mind
that all question types in CR like strengthen, weaken, evaluate, paradox etc. all
have in the question, which of the following statements if true…….

gmat.crackverbal.com
CrackVerbal Critical Reasoning Guide|73

PRACTICE QUESTIONS: STRENGTHEN AND WEAKEN

1. A sudden surge in the rate of reported cases of a certain disease in Aquaville


has resulted in investigations that revealed that in the past year, the level of
pollutants in Aquaville’s freshwater source has increased manifold. Medical
researchers have hypothesized that this polluted water is responsible for the
increased incidence of this disease.

Each of the following provides additional support for the researchers’ hypothe-
sis EXCEPT:

(A) Towns with geographic and demographic characteristics similar to those of


Aquaville, but without polluted water, have much lower rates of this disease.
(B) The symptoms of this disease, which help in its diagnosis, first start appear-
ing fifteen to eighteen months post infection.
(C) The same types of pollutants that have been found in Aquaville’s water have
been independently linked to occurrences of high rates of this disease in other
towns.
(D) The rate of new cases of diseases among Aquaville’s residents who pur-
chase bottled water from other sources is not unusually high.
(E) Most of the people afflicted with new cases of the disease had regularly
consumed unfiltered water from Aquaville’s water source.

gmat.crackverbal.com
CrackVerbal Critical Reasoning Guide|74

2. Automobile industry representative: New innovations in automobiles will


not happen unless the heavy development costs we incur can be recouped in
later sales. Therefore, the current 20 years of protection provided by patents
for all industries should be extended in the case of newly developed
automobile innovations.

Critic: In most other industries, new product development has happened


despite high development costs. So, such an extension is unnecessary.

Which of the following, if true, most strongly supports the automobile indus-
try’s argument against the challenge made above?

(A) The heavy electrical industry, which has development costs comparable to
those in the automobile industry, has not seen new innovations in recent years.
(B) Automobile companies are legally permitted to bring to market alternative
innovations that are sufficiently dissimilar to the patented innovation.
(C) Many recent industrial innovations have happened in products, for example,
in the computer and electronics industries, for which patent protection is often
very ineffective.
(D) Beta testing of the automobile innovation, which occurs after the patent is
granted and before the new innovation can be marketed, often takes as many
as 10 years to complete.
(E) There are several industries with much innovation in which the ratio of
research and development costs to revenues is higher than it is in the automo-
bile industry.

gmat.crackverbal.com
CrackVerbal Critical Reasoning Guide|75

3. Birds that gather at the private airway of a huge corporation pose a huge
problem for executives who use the corporate jets. These birds are known to fly
into the jet propellers and, as a consequence, cause the jets to crash. To solve
the problem, the corporation plans to import a large number of herding dogs to
keep the birds away from the jets.

Which of the following, if a realistic possibility, would cast the most serious
doubt on the prospects for success of the corporation’s plan?

(A) The dogs will need to be regularly taken away for vaccinations and quaran-
tine.
(B) The dogs have been trained to herd cattle and will need to be retrained to
herd birds.
(C) The dogs will chase away other small animals that live around the airway,
rousing criticism from nature activists.
(D) Herding dogs tend to form an uncontrollable pack in large numbers.
(E) Some of the birds will move to the neighboring airway in order to avoid
being herded by the dogs.

gmat.crackverbal.com
CrackVerbal Critical Reasoning Guide|76

4. Woodsville, not Aquaville, should build wind farms in its city. A wind farm,
which generates electricity using arrays of thousands of wind-powered
turbines, requires vast expanses of open land. Aquaville and Woodsville have
similar terrain, but the population density of Aquaville is significantly higher
than that of Woodsville.

Which of the following, if true, most seriously weakens the argument?

(A) Aquaville and Woodsville are adjacent to each other and both are located in
the windiest area of the country.
(B) The total land area of Woodsville is substantially greater than that of Aq-
uaville.
(C) Nearly all of Aquaville’s population is concentrated in a small part of the
county, while Woodsville’s population is spread evenly throughout the country.
(D) Wind farms require more land per unit of electricity generated than does
any other type of electricity generation facility.
(E) Some of the electricity generated by wind farms in Woodsville would be
purchased by users outside the county.

gmat.crackverbal.com
CrackVerbal Critical Reasoning Guide|77

5. The difference in price between big-brand jeans and less expensive


local-store brand jeans has become so wide that consumers have been
switching increasingly to store brand jeans despite the big brands’
reputation for better quality. To attract these consumers back, several
manufacturers of big-brand jeans plan to narrow the price gap between their
jeans and store brands to less than what it was five years ago.

Which of the following, if true, most seriously calls into question the likelihood
that the manufacturers’ plan will succeed in attracting back a large percentage
of consumers who have switched to store brand jeans?

(A) There is no significant difference among manufacturers of big-brand jeans


in the prices they charge for their products.
(B) Consumers who have switched to store-brand jeans have generally been
satisfied with the quality of those jeans.
(C) Many consumers would never think of switching to store-brand jeans be-
cause they believe the big-brand jeans to be of better quality.
(D) Because of lower advertising costs, stores are able to offer their own brands
of jeans at significantly lower prices.
(E) Total annual sales of jeans –including both big-brand and store-brand jeans
–have not increased significantly over the past five years.

gmat.crackverbal.com
CrackVerbal Critical Reasoning Guide|78

6. Woman magazine decided to change its image from being a family magazine
to concentrating on sex and gossip, thus appealing to a different readership.
Some advertisers subsequently withdrew their advertisements from Woman.
This must have been because they morally disapproved of Woman’s publication
of scandalous material.

Which one of the following, if true, provides the most support for this conclu-
sion?

(A) The advertisers switched their advertisements to other family magazines.


(B) Some advertisers switched from family magazines to advertise in the
changed Woman.
(C) The advertisers knew that their product sales would have increased more if
they had stayed on with Woman than if they had switched to other magazines.
(D) People who generally read family magazines are not likely to buy maga-
zines that concentrate on sex and gossip.
(E) Some of Woman magazine’s existing readers may disapprove of its new
image and content and would be likely to switch to other magazines.

gmat.crackverbal.com
CrackVerbal Critical Reasoning Guide|79

7. An increase in death rate from pulmonary tract infection in the past five
years has been blamed on the increase in urban pollution. However, since the
death rate from bronchitis has increased dramatically even in cities with little
or no pollution, one must instead conclude that the cause of increased deaths is
the use of the nebulizer, a device used to convert medication into inhalable
aerosol, by bronchitis sufferers to relieve their symptoms.

Each of the following, if true, provides support to the argument EXCEPT:

(A) Urban population has not shown any significant increase in the past five
years.
(B) The recording of deaths due to bronchitis has become more widespread and
accurate in the past five years.
(C) Evidence suggests that nebulizers make the lungs more vulnerable to repeat
attacks of bronchitis.
(D)By temporarily relieving the symptoms of bronchitis, nebulizers encourage
sufferers to avoid more beneficial long-term measures.
(E) Nebulizers gained widespread popularity as a treatment aid for bronchitis
only in the past five years.

gmat.crackverbal.com
CrackVerbal Critical Reasoning Guide|80

8. Boa constrictors have tiny hind leg bones buried in muscles toward their tail
ends. These vestigial legs are a clue that snakes descended from lizards over a
100 million years ago and walked the earth on their hind legs. A recently
discovered fossil of a boa constrictor has very fragile hind legs with
half-formed hind leg bones that could not have supported the animal’s weight
on land.

Which of the following statements, if also true, would most strongly support
the conclusion that the fragile hind leg bones are remnants of legs that boa
constrictors once had?

(A) No other snakes comparable to boa constrictors in size and weight have
remnants of hind leg bones.
(B) No fossils of ancient boa constrictors with intact hind legs capable of sup-
porting the animal’s weight have ever been discovered.
(C) It has not yet been conclusively proved that these vestigial legs of boas are
non-functional.
(D) Older boa constrictor fossils confirm that ancient boas had full hind leg
bones.
(E) Animals with hind legs would typically also have pelvises and these would
be evident in their fossils.

gmat.crackverbal.com
CrackVerbal Critical Reasoning Guide|81

9. Burning leaves and domestic refuse on open ground releases high levels of
carbon monoxide and carbon dioxide. Therefore, legislation is needed to
regulate the burning of leaves and other refuses on open ground.

Which of the following, if true, provides the most support for the argument
above?

(A) The amount of carbon monoxide released when leaves are burnt is much
less than the amount contained in an equal volume of automobile exhaust.
(B) The presence of large quantities of carbon monoxide and carbon dioxide in
the atmosphere negatively impacts air quality and therefore, human health.
(C) The harmful effects of carbon dioxide and carbon monoxide in the atmos-
phere have not yet been proven conclusively.
(D) No significant beneficial effect on air quality would result if open-air fires
were banned within the jurisdiction covered by the proposed legislation.
(E) Within the jurisdiction covered by the proposed legislation, most waste
disposal is done in non-polluting indoor incinerators.

gmat.crackverbal.com
CrackVerbal Critical Reasoning Guide|82

10. Cafe Cappuccino Pvt Ltd owns almost all coffee shops in Borodia and has
announced plans to double the number of coffee shops it has in the country in
the next five years. Yet, the number of people who walk into cafe Cappuccino is
only just large enough for profitability now and the country’s population in not
expected to increase drastically over the next ten years due to the
government’s strategic measures taken to control population explosion.
Clearly, therefore, if there is indeed no increase in population, Cafe
Cappuccino’s new outlets are unlikely to prove profitable.

Which of the following, if true about Borodia, most seriously weakens the
arguments?

(A) Though very little change in the size of the population is expected, a pro-
nounced shift towards a younger, more affluent, and more entertainment
oriented population is expected to occur.
(B) The sales of ice creams account for more of Cafe Cappuccino’s profits than
the sales of coffee and tea.
(C) In selecting the mix of items that are served at Cafe Cappuccino the policy is
to avoid those that appeal to only a small segment of the cafe going popula-
tion.
(D) Spending on confectionary as well as on homemade chocolates is currently
no longer increasing.
(E) There are no population centers in the country that are not already served
by at least one of the coffee shops that Cafe Cappuccino owns and operates.

gmat.crackverbal.com
CrackVerbal Critical Reasoning Guide|83

ANSWER EXPLANATIONS

1. Answer: B

The conclusion is that the disease spike is caused by the polluted water source.
You should now pick an answer choice that doesn’t support the researcher’s
findings or doesn’t affect the argument at all.

(A) It provides supporting evidence by saying that places without this water
source has lesser occurrence of the disease.
(B) This doesn’t affect the argument in any way. Therefore, it is the correct
answer.
(C) It provides supportive evidence by linking the disease and the polluted
water.
(D) It provides supportive evidence by saying that water from other sources
does not cause the disease.
(E) It provides supportive evidence by saying that people without the disease
consumed water from some other water source.

gmat.crackverbal.com
CrackVerbal Critical Reasoning Guide|84

2. Answer: D

To counteract the argument that the patent should not be extended, we need
proof that suggests that an extension is necessary. Only option D provides any
relevant information with respect to the argument. If it takes 10 years post
patenting to test a product before marketing it, it could be true that an exten-
sion may be required.

(A) There could be other reasons for why the heavy electrical industry does not
see innovation — we cannot assume this is because of the duration of patent
protection.
(B) So what? The same cost considerations will apply to these innovations too.
(C) It implies that patent protection isn’t really necessary for innovations. It
goes against what the automobile industry representative is trying to argue.
(D) This answer choice supports the automobile industry representative’s
argument.
(E) It says that other industries with a higher R&D to revenues ratio can inno-
vate. It goes against the automobile industry representative’s argument.

3. Answer: D

The assumption is that this plan of getting herding dogs to herd the birds will
work. Options A, B, C, and E do not introduce any evidence to suggest that the
plan won’t work.

(A) So what? Not all the dogs need to be taken away at the same time. ’Regular’
could also mean annual or biennial.
(B) So what? If they need to be trained, they can be trained
(C) So what if nature activists criticize the corporation?
(D) This could make the plan non-viable, since a herd of dogs that is uncontro-
lable will result in the bird control being ineffective.
(E) This will aid the plan, not weaken it.

gmat.crackverbal.com
CrackVerbal Critical Reasoning Guide|85

4. Answer: C

The conclusion is that Woodsville should build the wind farms, and not
Aquvaille. The reason provided is that wind farms require a large open space
and Aquaville has higher population density. The assumption is that large
population density is indicative of lack of vast open spaces.

(A)“So what?”. This option is totally irrelevant.


(B) This doesn’t tell anything about the availability of open spaces.
(C) This disproves the assumption above and hence is the correct answer.
(D) This is an irrelevant answer choice.
(E) This doesn’t relate to the argument in any way.

5. Answer: B

The argument states that reducing prices would bring back customers for
big-brand jeans manufacturers. The assumption is that it is only the price that
has attracted those customers who have purchased store-brand jeans. Option B
questions this assumption and effectively weakens the argument.

(A) Differences in prices charged by manufacturers is irrelevant.


(B) If this is true, there is no reason for consumers to switch.
(C) Consumers who would not switch to store brands in any situation are irrel-
evant to this argument.
(D) Perhaps stores can significantly lower their prices —but how much lower
isn’t indicated.
(E) This is not relevant to the argument.

gmat.crackverbal.com
CrackVerbal Critical Reasoning Guide|86

6. Answer: C

The assumption made here is that the withdrawal of some advertisers from
Woman was not for any other reason other than that of moral disapproval.
Option C shows that withdrawing from Woman will prove to be disadvanta-
geous monetarily to these advertisers, thereby eliminating monetary incentives
as cause for the withdrawal, making “moral reasons” more probable a reason.

(A) Yes, the advertisers moved to other family magazines —but for what rea-
son? It is not clear that morality was the reason.
(B) Those who moved to Woman are irrelevant to this argument.
(C) This is the right answer as described above.
(D) This is generic statement and is irrelevant to the argument.
(E) So what? How many is ’some’?

7. Answer: B

The argument says that the increase in death rate due to bronchitis in the past
five years is solely due to the use of nebulizers. That means other causes must
be ruled out. Since this is an ‘except’ question, we need to identify and elimi-
nate all the choices that strengthen this conclusion.

(A) This eliminates population increase as an alternative cause for an increase


in the death rate.
(B) This does not support the argument, and hence, it is the correct answer.
(C) This is a negative effect of nebulizers.
(D) This is a negative effect of nebulizers.
(E) This adds strength to why nebulizers could be a likely cause as they’ve been
in widespread use only in the past five years.

gmat.crackverbal.com
CrackVerbal Critical Reasoning Guide|87

8. Answer: D

As the question points out, the conclusion of the argument is that the “fragile
hind leg bones are remnants of legs that boa constrictors once had.” The rest of
the argument provides supporting data that help strength the conclusion.

(A) The features of other snakes is irrelevant to the argument.


(B) Fossils with intact hind legs aren’t necessary to offer proof that boas had
hind legs
(C) Whether the legs are functional or not is irrelevant.
(D) This supports the argument, and hence, it is the correct answer.
(E) So what? We don’t know whether the boa fossil also had a pelvis.

9. Answer: B

The conclusion of the argument is that there is a need for a legislation to regu-
late the burning of leaves and other refuses on open ground. The reason why
the author comes to this conclusion is that the burning leaves and domestic
refuse on open ground releases high levels of carbon monoxide and carbon
dioxide.

The assumption of the argument is that the release of carbon monoxide and
carbon dioxide is the reason for the need of the legislation. We will need an
answer choice that will support the above assumption.

(A) Comparison between the quantities of carbon monoxide released when


leaves are burnt vis-à-vis through automobile exhaust is irrelevant.
(B) If the presence of these gases in the atmosphere negatively impacts health,
then legislation to regulate this is necessary. This strengthens the conclusion,
and is therefore the correct answer.
(C) This weakens the argument.
(D) This weakens the argument.
(E) It does not matter. The legislation is relevant as long as the practice of
burning leaves and refuse in open ground continues.
gmat.crackverbal.com
CrackVerbal Critical Reasoning Guide|88

10. Answer: A

The conclusion of the argument is that Cafe Cappuccino’s new outlets will not
prove profitable if there is no increase in population. The various reasons given
to support the above conclusion are that there are measures to control the
population in that particular country and the number of people who walk in to
Cafe Cappuccino’s is just large enough for profitability now.

Now, we need to find the option that would go against the above argument, i.e.,
the argument that tells you that the sales at Cafe Cappuccino would not in-
crease. You should be finding on option that says that there would be an in-
crease in sales despite all the control measures.

(A) This option goes against the argument by saying that the sales will not
increase.
(B) It doesn’t tell you that Cafe Cappuccino will acquire profits.
(C) It doesn’t affect the conclusion.
(D) This option talks about some other type of product, and we don’t know
whether it is sold in café Cappuccino or not.
(E) This option also doesn’t affect the fact that Cafe Cappuccino will acquire
profits.

gmat.crackverbal.com
CrackVerbal Critical Reasoning Guide|89

EXCEPT QUESTIONS

Sometimes the GMAT will make a question more complex by using the word
except. Any question type can be manipulated using the “except” formulation.
You will be given certain instructions and asked to pick an answer choice that
does not conform to the criteria given.

How should you approach such questions?

Rephrase the “except” statement into a question, inserting the word “not” and
eliminating the word “except”. This will make it easier to process.

Do remember that “except” does not mean ’find the opposite’. For example, if
the question says “All the statements below strengthen the argument except…”
then the correct answer choice could weaken the argument, or be completely
irrelevant.

gmat.crackverbal.com
CrackVerbal Critical Reasoning Guide|90

gmat.crackverbal.com
CrackVerbal Critical Reasoning Guide|91

5. Flaw and Evaluate questions types

• Type 4: Flaw in the Argument


• Type 5: Evaluate the Argument
• Practice Questions: Flaw and Evaluate

gmat.crackverbal.com
CrackVerbal Critical Reasoning Guide|92

gmat.crackverbal.com
CrackVerbal Critical Reasoning Guide|93

TYPE 4: FLAW IN THE ARGUMENT

Such questions ask you to point out a flaw/defect/error/fallacy inherently


present in the argument. This question type is a variant of the weaken question.
However instead of providing an external premise to weaken, there is some-
thing inherently wrong with the conclusion that candidates have to identify.
Example:

John: “Dad, it’s not my fault that I failed the math test; my math teacher, Mr.
John Wilkes has never liked me. He failed me because of this.”

Do you think the argument is flawed?

The flaw in the child’s argument is this: he fails to realize that someone other
than Mr. John Wilkes could have evaluated the paper!

Illustrative example

Men aged between 40 and 45 are more likely to purchase tobacco based prod-
ucts and are more likely to purchase them in larger amounts than are members
of any other demographic group. Therefore the popular belief that teenagers
consume more tobacco based products than adults must be false.

The argument is flawed primarily because the author

(A) Fails to distinguish between purchasing and consuming.


(B) Does not supply information about men of age groups other than 4o to 45.
(C) Depends on popular belief rather than on documented research findings.
(D) Does not specify the precise amount of tobacco purchased by any demo-
graphic group.
(E) Discusses tobacco products rather than more dangerous products such as
alcohol.

gmat.crackverbal.com
CrackVerbal Critical Reasoning Guide|94

What do you understand from the argument here?

The conclusion of the argument is that adult men consume more tobacco based
products than teenagers do, as opposed to the popular belief. He arrives at the
conclusion based on the fact that men aged 40 to 45 are more like to consume
tobacco based products and are more likely to purchase them in larger amounts
than are members of any other demographic group.

If you had noticed, the author makes a shift in language here by using the
words ”consuming” and ”purchasing” interchangeably. That is the major flaw in
the argument above.

(A) It points this flaw very explicitly.


(B) We don’t really need this information.
(C) The author gives us information.
(D) Do we really need to know the amount of tobacco based products?
(E) Again, do we really need to know about alcohol?

TYPE 5: EVALUATE THE ARGUMENT

Such questions require test-takers to evaluate the validity of a given conclusion


or to suggest a way in which one could efficiently evaluate a given conclusion.
While tackling these, you need to consider the assumptions upon which the
argument is based. You need to think like a decision-maker while answering
such questions. Ask yourself what additional information you will need to
evaluate the validity of the conclusion.

Example:

For the previous example, if we had the additional information, “the test papers
are evaluated by a computerized grading machine,” then we can definitely take
a call on whether the argument is valid (here this information suggests that the
argument is not valid).

gmat.crackverbal.com
CrackVerbal Critical Reasoning Guide|95

This piece of information could be presented in another way:

“Whether the test papers were evaluated by a computerized grading machine”

In such cases, where the answer option provides a question statement, consider
the possible outcomes.

Here if the answer to the question statement is “yes”, the argument as a whole
is weakened. On the other hand, if the answer to this statement is “no”, the
argument is strengthened. Therefore, when an answer option provides a ques-
tion statement —consider the responses; one of the responses will strengthen
the argument and the other will weaken the argument.

Illustrative example

The shaft in the gear box often becomes less responsive after a few weeks of
intense use. A researcher, whose son is a biker, hypothesized that dirt and oil
rather than changes in the material properties of the shaft were responsible.

Which of the following information is most likely to yield significant infor-


mation that would help evaluate the researcher’s hypothesis?

(A) Determining if a metal alloy is used to make the shaft used by bike racers
(B) Determining whether mountain bikers make their gear-shafts become less
responsive faster than do normal bikers
(C) Determining whether identical lengths of shafts, of the same dimensions,
become less responsive at different rates when used in various brands of bikes
(D)Determining whether a dead shaft and a new shaft are able to produce the
same RPM
(E) Determining whether smearing various substances on new guitar strings
causes them to become less responsive

gmat.crackverbal.com
CrackVerbal Critical Reasoning Guide|96

What do you understand from the argument here?

The given argument says that the shaft becomes less responsive because of oil
and dirt. We will now need to find a parameter that will help us understand this
scenario.

(A) We don’t need to know the material with which the shaft is made.
(B) We don’t need to know what kind of bikers use the shaft.
(C)We don’t need to know the brand of bikes.
(D) We don’t need to know the correspondence between RPM and the type of
shaft.
(E) The answer to this question tells you whether the dust has any effect on the
responsiveness of the guitar strings.

gmat.crackverbal.com
CrackVerbal Critical Reasoning Guide|97

PRACTICE QUESTIONS: FLAW AND EVALUATE

1. The current proposal to give university students a broader choice in planning


their own courses of study should be abandoned. The students who are sup-
porting the proposal will never be satisfied, no matter what requirements are
established. Some of these students have reached their third year without
declaring a major. One first-year student has failed to complete four required
courses. Several others have indicated a serious indifference to grades and
intellectual achievement.

A flaw in the argument is that it does which one of the following?

(A) Avoids the issue by focusing on supporters of the proposal


(B) Argues circularly by assuming the conclusion is true in stating the premises
(C) Fails to define the critical term “satisfied”
(D) Distorts the proposal advocated by opponents
(E) Uses the term “student” equivocally

gmat.crackverbal.com
CrackVerbal Critical Reasoning Guide|98

2. When people who have not used ketamine, a narcotic drug, are tested for
ketamine use, on an average, only 5 will test positive for every 100 tested. By
contrast, of every 100 people who have used ketamine, 99 will test positive.
Thus, when a randomly chosen group of people is tested for ketamine use, the
vast majority of those who test positive will be people who have used
ketamine.

A reasoning error in the argument is that the argument


(A) Attempts to infer a value judgment from purely factual premises.
(B) Attributes to every member of the population the properties of the average
member of the population.
(C) Fails to take into account what proportion of the population have used
ketamine.
(D) Ignores the fact that some ketamine users do not test positive.
(E) Advocates testing people for ketamine use when there is no reason to
suspect that they have used ketamine.

gmat.crackverbal.com
CrackVerbal Critical Reasoning Guide|99

3. Working professionals aged between 35 and 45 are more likely to purchase


the latest gadgets and are more likely to purchase it in larger numbers than are
teenagers or young adults. Therefore the popular belief that teenagers and
young adults spend more time with smart-phones, tablets, and other gadgets
must be false.

The argument is flawed primarily because the author

(A) Does not specify the precise number of gadgets purchased by any demo-
graphic group.
(B) Discusses gadgets rather than more widely utilized tools.
(C) Depends on popular belief rather than on documented research findings.
(D) Fails to distinguish between “purchasing” and “spending time with”.
(E) Does not supply information about working professionals aged 35 to 45.

gmat.crackverbal.com
CrackVerbal Critical Reasoning Guide|100

4. Approximately 8 million income-earning men have children who are be-


tween 3-5 years of age, and approximately 7 million men are the sole income
earners for their families. These figures indicate that there are comparatively
few income-earning men who have children belonging to the age group of 3-5
but are not the sole income earners for their families.

A major flaw in the reasoning is that it

(A) Ignores the possibility, that families with children belonging to the age
group of 3-5 might also have older children.
(B) Fails to indicate whether the difference between the two figures cited will
tend to remain stable over time.
(C) Relies in figures that are too imprecise to support the conclusion drawn.
(D) Overlooks the possibility that there is little or no overlap between the two
populations of men cited.
(E) Provides no information on families in which women are the sole income
earners.

gmat.crackverbal.com
CrackVerbal Critical Reasoning Guide|101

5. Selina: All the campers at Camp Minnesota go to Tri-State High School.


Bruce: That’s not true. Some Tri-State students are campers at Camp Lakeview.

Bruce’s answer can be best explained on the assumption that he has inter-
preted Selina’s remark to mean that

(A) Most of the campers at Camp Lakeview come from high schools other than
Tri-State High School.
(B) Only campers at Camp Minnesota are students at Tri-State High School.
(C) Some Tri-State High School students have withdrawn from Camp Lakeview.
(D) All Tri-State High School students have withdrawn from Camp Lakeview.
(E) Most Tri-State High School students are campers at Camp Minnesota.

gmat.crackverbal.com
CrackVerbal Critical Reasoning Guide|102

6. Scientists have modified salmon embryos genetically to increasing the


embryo’s hardiness and longevity when it grows into an adult salmon. Fish
harvesters, those who grow and sell fish domestically, who tried out the genet-
ically modified salmon embryos last year used and therefore spent less on
vaccinations and medication for the fish and still got a yield (healthy fish meat
that can be processed and sold as canned food) comparable to that they had
got with ordinary salmon. Ordinary embryos, however, cost less, and what these
fish harvesters saved on vaccines and medication was exceeded by the extra
costs of the genetically engineered embryos. Therefore, for most fish harvest-
ers, switching to genetically modified salmon embryos would be unlikely to
increase profits.

The answer to which of the questions would help evaluate the conclusion that
the profits are unlikely to increase.

(A) Whether there are diseases that sometimes reduce fish yield, but against
which commonly used medicines, vaccination, and genetic modification are
equally ineffective
(B) Whether the price that fish harvesters receive for fish yield for salmon has
remained steady over the past few years
(C) Whether the vaccinations and medication typically used on fish tend to be
more expensive than those typically used on other fish types
(D) Whether most of the fish harvesters who tried the genetically modified
salmon embryos last year used more vaccination and medication than was
actually necessary
(E) Whether it is the most profitable fish species for most fish harvesters who
grow salmon

gmat.crackverbal.com
CrackVerbal Critical Reasoning Guide|103

7. A certain health supplement manufacturer claims that its dietary supple-


ment, Paxol, is highly effective in helping people achieve physical fitness and
improve endurance. The company supports this claim by providing the results
of a recent experiment that involved over 500 subjects who were physically
unfit (obese, high blood sugar etc). During the 9-week experiment, each of the
subjects was required to take a regular dose of Paxol every day immediately
after their daily 2-hour workouts supervised by a professional fitness instruc-
tor. As a result of the experiment, the subjects lost an average of 12 pounds of
weight per person, and over 95% of all participants demonstrated higher
physical endurance.

The answer to which of the following questions would be most helpful in eval-
uating the conclusion of the health supplement manufacturer?

(A) What was the daily dose of Paxol that the subjects were required to take?
(B) What was the maximum weight lost by any participant during the 9-week
program?
(C) What was the average age of the subjects participating in the experiment?
(D)Did the majority of subjects experience a significant improvement in physi-
cal strength?
(E) What would be the average weight loss and the improvement in endurance
in a group of subjects with similar characteristics involved in the same physical
fitness program but not taking Paxol?

gmat.crackverbal.com
CrackVerbal Critical Reasoning Guide|104

8. A rare type of bacterial infection has killed off many tuna fish in North Amer-
ica; fishing boats have had a very meager catch of tuna this year. As a result the
wholesale price of tuna meat has increased significantly. So the retail price of
tuna-based cat food is certain to increase within six months.

The answer to which of the following questions would provide information


relevant to evaluating the claim made in the argument above?

(A) Can the bacterial infection be eliminated within the next six months?
(B) Will consumers resort to buying tuna meat wholesale in the next six
months?
(C) Has the price of cat food remained steady during other periods of poor tuna
catches?
(D) What percentage of tuna in North America were affected by the bacteria?
(E) Have the prices of other ingredients in tuna-based cat food decreased
recently?

gmat.crackverbal.com
CrackVerbal Critical Reasoning Guide|105

9. Magnavox, a company that makes gaming consoles and games, has seen,
through the years, declining profits for its flagship product, GameBox 1. The
CFO of Magnavox investigated the issue and determined that an increase in
chip costs had raised the cost of producing the console but consumers who
were surveyed had reported that they weren’t willing to pay more than the
lower price that GameBox 1 was selling at the time of its launch. As a result, the
CFO recommended that the company stop producing this product because the
CEO wanted to sell only products whose profit margins were increasing.

The answer to which of the following questions would be most useful in evalu-
ating whether the CFO’s decision to divest the company of its flagship product
is warranted?

(A)Does the company have new and profitable products available with which to
replace the flagship product?
(B)Is there a way to alter the manufacturing or distribution processes in order to
reduce the cost to produce the flagship product?
(C) How will Magnavox’s brand name be impacted if they stopped producing
GameBox 1?
(D) Are there additional features which could be added to the product and for
which consumers might be willing to pay a higher price?
(E) What percentage of Magnavox’s revenues is represented by the sales of the
flagship product in question?

gmat.crackverbal.com
CrackVerbal Critical Reasoning Guide|106

10. The manager of BigShoppe, a company that lets customers purchase fresh
fruits and vegetables online, suggests that customer orders be packed in ex-
tra-strong cartons to eliminate damage during transportation. Though these
cartons are more expensive, overall costs would remain unaffected since the
additional packaging cost is almost equal to the cost of refunding customers
who received damaged fruits and vegetables.

Which of the following would it be most important to ascertain in determining


whether implementing the manager’s proposal would have the argued-for
effect on costs?

(A) Whether BigShoppe dispatchers habitually send out damaged fruits and
vegetables to customers
(B) Whether the transportation companies used by BigShoppe will handle the
new cartons carefully
(C) Whether the percentage of refunds that BigShoppe makes is higher than
those seen by other online shopping companies
(D) Whether some customers are themselves responsible for the damage
caused to the produce
(E) Whether fruits and vegetables are more vulnerable to incurring damage
during shipping than are other fragile products.

gmat.crackverbal.com
CrackVerbal Critical Reasoning Guide|107

ANSWER EXPLANATIONS

1. Answer: A

The argument put forward is flawed because it counters the proposal not by
addressing the demerits of the proposal itself, but by analyzing the student
population who voted for the proposal instead. This is best expressed in option
A.

(A) This option points out the flaw mentioned above.


(B) It doesn’t argue circularly.
(C) There is no need to define the term “satisfied”.
(D) It doesn’t distort the proposal.
(E) There is no need to define the term “student” and that too equivocally.

2. Answer: C

Assume that there are no users of ketamine in the population. The 5 people
(statistically) who test positive will then be false positives. The test result
would then be inaccurate. Therefore it is essential to know the proportion of
ketamine users vs non-users to know how effective the test will be. Option C
expresses this perspective.

(A) No inference is being made here.


(B)No generalizations are made here.
(C) This option points out the flaw mentioned above.
(D) This doesn’t happen in the passage.
(E) This doesn’t help the argument in any way.

gmat.crackverbal.com
CrackVerbal Critical Reasoning Guide|108

3. Answer: D

The flaw in the argument is the language shift between “purchasing” and
“spending time with.” This is established by option D.
(A) We don’t need to know the number of gadgets.
(B) There is no need to discuss gadgets.
(C)Again, there is no need to learn about documented research findings.
(D)This option points out the flaw mentioned above.
(E) We don’t need information about working professionals.

4. Answer: D

The argument here talks about 8 million and 7 million people, but it doesn’t
realize that the 7 million is NOT a subset of the 8 million. Therefore the claim
that this difference is “comparatively few” is wrong. Option D, therefore, is
correct.

(A)This doesn’t talk about the subset aspect.


(B) The number doesn’t have to remain stable.
(C) The number is not imprecise.
(D) This option points out the flaw mentioned above.
(E) We don’t need information about women being the sole income earners.

gmat.crackverbal.com
CrackVerbal Critical Reasoning Guide|109

5. Answer: B

Selina’s argument is that ALL X go to Y. Bruce misunderstands this to mean that


all Y belong to X. Option B elucidates this assumption that Bruce made.

(A) This goes against the argument.


(B) This option points out the flaw mentioned above.
(C) This cannot be the assumption.
(D) There is no information in the argument.
(E) There is no information to support this option.

6. Answer: D

The argument states that using the genetically altered embryos will not be
profitable to most fish harvesters. The assumption is that this is necessarily
true. Option D poses a relevant question. If it is found that fish harvesters did
use more vaccine and medication than was necessary, they were spending too
much on those than required. This weakens the argument. On the other hand, if
the fish harvesters weren’t using more vaccines and medication than required,
it would strengthens the argument that using genetically modified embryos
will not be lucrative. Thus, the answer to option D helps us evaluate the argu-
ment. Hence, it is correct.

(A) The answer to this question doesn’t help us decide whether we need medi-
cation.
(B)We don’t need to know whether the price has remained steady or not.
(C) We don’t need to know whether the medicine is expensive.
(D) This option helps us evaluate the argument as mentioned above.
(E) Again, it doesn’t matter if we know whether the given fish harvest is
profitable or not.

gmat.crackverbal.com
CrackVerbal Critical Reasoning Guide|110

7. Answer: E

The questionable link in this argument is that of causation. If it were found that
people who did not take Paxol but participated in the fitness program lost as
much weight and increased as much endurance as the group that took the
supplement, then it weakens the argument that Paxol is responsible for the
improvement in physical fitness and endurance. On the contrary if people who
don’t take the supplement do not scale as well physically as the other group,
then it strengthens the argument that Paxol was the cause of improved fitness.
As the answer to E helps us evaluate the argument, it is the correct option.

(A) We need to know whether the drug is effective in achieving physical fitness
and not about the dosage.
(B) Again if there is a minimal weight loss, it could still be towards achieving
physical fitness.
(C) We don’t need to know the average age to determine whether there would
be physical fitness.
(D) We don’t have to be worried about improvement in physical strengths. It is
about physical fitness.
(E) This option helps us evaluate the argument as mentioned above.

8. Answer: E
The argument assumes that the increase in tuna price will invariably result in an
increase in the price of tuna based cat food. This may not be true if the other
ingredients’ prices dropped. This is established by option E and is the correct
answer.

(A) We don’t need to know this in order to determine the information about the
price.
(B)We don’t need to know whether people will buy the meat.
(C) This could be a contender but still this doesn’t answer question about the
price.
(D) This doesn’t answer the question about the price of the meat.
(E) This option helps us evaluate the argument as mentioned above.
gmat.crackverbal.com
CrackVerbal Critical Reasoning Guide|111

9. Answer: B

The argument states that the product be stopped to satisfy the CEO’s require-
ment that only products with increasing profit margins be sold. The only reason
for this is that the product costs more and people aren’t willing to pay more for
the product. If, however, the cost could be reduced, the CEO’s goal could be
met without doing something as drastic as withdrawing the company’s flagship
product. Option B introduces exactly this perspective.

(A)This doesn’t contribute to the CFO’s goal.


(B) This option helps us evaluate the argument as mentioned above.
(C) We don’t need to know the impact.
(D) We don’t need to know about the effect that the additional features have.
(E) We don’t need to know the percentage of sales.

10.Answer: A

The argument is that using superior packaging would be beneficial and will
offset the cost that BigShoppe incurs through refunds. The assumption is that
using superior packaging would essentially ensure that the fruits and vegeta-
bles BigShoppe sends to customers will not have to be replaced. Option A
poses the right evaluation, in that if it is found that BigShoppe is sending
already damaged fruits and vegetables to customers, then the damage isn’t
caused during transportation and therefore, using better packing material
would be pointless. Thus, A is the correct answer.

(A) This option helps us evaluate the argument as mentioned above.


(B)We are worried about the superior packaging. We don’t need to know about
how they will be handled.
(C) We don’t need to know about the percentage of refunds.
(D) We don’t need to know who is responsible.
(E) We don’t need to know about other products.

gmat.crackverbal.com
CrackVerbal Critical Reasoning Guide|112

gmat.crackverbal.com
CrackVerbal Critical Reasoning Guide|113

6. Content-based CR questions

• How are Content-Based Questions Different from Assumption-Based


Questions?
• Type 1: Inference/Conclusion/Main Point Questions
• 4 Golden Rules for Content-based questions
• Practice Questions: Inference/Main Point/Conclusion
• Type 2: Resolve the Paradox
• Practice Questions: Resolve the Paradox

gmat.crackverbal.com
CrackVerbal Critical Reasoning Guide|114

gmat.crackverbal.com
CrackVerbal Critical Reasoning Guide|115

HOW ARE CONTENT-BASED QUESTIONS DIFFERENT FROM ASSUMPTION-BASED


QUESTIONS?

For content-based questions, the only important parts of the questions are
conclusions and premises! These questions are based on what has been stated
rather than what is unstated. After identifying a question as a content-based
question, there is no need to consider the assumptions made.

While attempting such questions, it is very important to understand the written


word well. You need to translate the text in your own words. Think of yourself
as a lawyer and try to see how well you can read the “legalese”! Some students
find it helpful to put down their understanding of such questions in words or as
diagrams.

What is an Inference?

An inference is any piece of information that must be true, given another body
of information. There is no speculation in an inference; it is an inevitable,
indubitable conclusion arrived at from the information given. A valid inference
must be absolutely positively true beyond any question or doubt.

How do you make any statement always true, under any circumstance?
By introducing a broad term such as ’perhaps’ or ’maybe’! For instance, What is
the color of the sky? It is perhaps blue.

The beauty of such an answer is that it is very hard to disprove. If someone


points out that the color of the sky is not really blue, the respondent can say
that he had only said that it was ’perhaps blue’.

Thus, by broadening its scope, any statement has a lesser chance of being
disproved. When you look at inference questions on the GMAT, be very wary of
choices that make absolute or blanket statements. On the other hand, state-
ments that have caveats such as perhaps, maybe, probability, likelihood, etc.
are more likely to be correct.

gmat.crackverbal.com
CrackVerbal Critical Reasoning Guide|116

TYPE 1: INFERENCE / CONCLUSION / MAIN POINT QUESTIONS

These questions test your ability to understand a set of facts without altering
them. You will have to use the 100% truth rule to tackle such questions. In
essence, the correct answer must be true. It is not enough that the answer
choice “may be true”. On such questions, there is absolutely no room for error.

The GMAT will try to trick you into believing what it said versus what it meant.
Remember the Mahabharata incident where Yudhishtira tricks Drona by saying
“Ashwathama is dead”? This is a classic example from mythology that helps us
understand content based questions.

For such questions eliminating incorrect answer choices would be a better


strategy than attempting to pick the right answer choice. One of the major
mistakes that the amateur GMAT test-taker makes on these questions is allow-
ing what he/she believes to be true to take precedence over what is known or
stated to be true.

4 GOLDEN RULES FOR CONTENT-BASED QUESTIONS

Rule 1: Never go far beyond the premise while looking for an inference.

Rule 2: Be very wary of answer choices that contain extreme words or phrases
such as always, never, or all.

Rule 3: Remember the litmus test: the correct answer should be 100% true,
always.

Rule 4: Use the process of elimination effectively to zero in on the right answer
choice

gmat.crackverbal.com
CrackVerbal Critical Reasoning Guide|117

Some Other “Flavors” of the Same Type

Which of the following conclusions/assertions/inferences/statements is most


strongly supported by the passage?
Which of the following must be true as per the passage?
Which of the following can be inferred from the passage?
The author is arguing that…
The main point of the passage is that
Which of the following can complete the passage?
Which of the following is an example of the … given in the passage?
All of the above require you to infer something based on the information given in
the question stem.

Illustrative example

In India, for mangoes to ripen, there must be mango showers along with optimal
temperatures for the process. Mango showers in the month of May are very
common and this leads to ripening of fruits, but such showers also lead to
reduction of temperature, below the optimal temperature level required for
mangoes to ripen.

The claims above, if true, most strongly support which of the following conclu-
sions?

(A) The climatic conditions in India usually support the growth of Mangoes.
(B) As long as the temperatures remain in the optimal range, mangoes will
definitely ripen in India.
(C) Mango showers in India would ensure that the temperatures don’t fall below
the optimal level.
(D) Any attempt to ripen the mangoes in India, is likely to fail sometime.
(E) For mango showers to occur, it is necessary that the temperature is main-
tained at an optimal level.

gmat.crackverbal.com
CrackVerbal Critical Reasoning Guide|118

This is an inference question. So it is best to go through the argument and note


down the facts that are given. Once that is done, you will need to find an option
that is best supported by the information given.

(A) No information about climatic conditions is mentioned.


(B)Such an extreme inference cannot be made.
(C)There is no information given to support this.
(D) Yes. Because no matter what attempt is taken, both the conditions wouldn’t
be prevalent at all.
(E)Again, there is no information to support this option.

gmat.crackverbal.com
CrackVerbal Critical Reasoning Guide|119

PRACTICE QUESTIONS: INFERENCE / MAIN POINT / CONCLUSION

1.Baking for winter holidays is tradition that may have a sound medical basis. In
midwinter, when days are short, many people suffer from a specific type of
seasonal depression caused by lack of sunlight. Carbohydrates, both sugars and
starches, boost the brain’s levels of serotonin, a neurotransmitter that is re-
sponsible for determining mood. In this respect, carbohydrates act on the brain
in the same way as some antidepressants do. Thus, eating holiday cookies may
be an effective remedy for winter depression.

Which one of the following can be properly inferred from the passage?

(A) Seasonal depression is one of the most easily treated forms of depression.
(B)Lack of sunlight lowers the level of serotonin in the brain.
(C) People are more likely to be depressed in midwinter than at other times of
the year.
(D) Antidepressants function by changing the brain’s level of serotonin.
(E) Raising the level of neurotransmitters in the brain effectively relieves
depression.

gmat.crackverbal.com
CrackVerbal Critical Reasoning Guide|120

2. Disturbed by growing obesity levels among citizens, the Mayor of Vargonia


donated cookware and kitchen appliances to schools and colleges in the
county that offered cooking classes. He wanted to promote better eating habits
by encouraging people to cook their own food at home. Quite a few citizens of
Vargonia have decided to vote for the Mayor in the upcoming elections.

Which of the following inferences is best supported by the passage given


above?

(A) Eating home-cooked food is the simplest way to combat obesity.


(B) Schools and colleges that offer cooking classes were affected by a lack of
cookware and kitchen appliances.
(C) Making donations is the best way to garner political support.
(D) Actions that benefit others sometimes have positive consequences for
those who perform them.
(E) The primary driver of citizens’ voting decisions is the donations that election
candidates make to public causes.

gmat.crackverbal.com
CrackVerbal Critical Reasoning Guide|121

3. There is little point in looking to poets for insights into political issues. Most
of them hold political views that are less insightful than those of any reasona-
bly well-educated person who is not a poet. Indeed, when taken as a whole, the
statements made by a poet, including those considered great, indicate that
poetic talent and political insight are rarely found together.

Which one of the following can be inferred from the passage?

(A)There are no poets who have insights into political issues.


(B) Some poets are no less politically insightful than some reasonably
well-educated persons who are not poets.
(C) Every reasonably well-educated person who is not a poet has more insight
into political issues than any poet.
(D)Politicians rarely have any poetic talent.
(E) An education in or taste for poetry makes a person reasonably well educat-
ed.

gmat.crackverbal.com
CrackVerbal Critical Reasoning Guide|122

4. Last year the domestic consumption of fresh water, water collected directly
from water sources such as rivers and ponds, in Aquaville was twice that of
recycled water, used water that has been purified and treated artificially. An
analyst has projected that by 2015, the city will use at least as much recycled
water annually as it does fresh water, while using a greater quantity of fresh
water than it did last year.

If the information above is correct and the analyst’s projections prove to be


accurate, which of the following projections must also be accurate?

(A) Aquaville will consume less water in 2015 than it did last year
(B) In 2015, Aquaville will use at least twice as much water, in total, as it did last
year.
(C) As compared with last year, in 2015, Aquaville will consume less recycled
water.
(D) In 2015, Aquaville will consume more water from a given supply of water
than it did last year.
(E) In 2015, Aquaville will use at least twice as much recycled water as it did
last year

gmat.crackverbal.com
CrackVerbal Critical Reasoning Guide|123

5. The gaming market recently went through a huge transformation; therefore,


Jinx Inc. must increase productivity by 10 percent over the course of the next
two years if it wants to avoid bankruptcy. However, Jinx’s production structure
is such that if a 10 percent productivity increase is possible, then a 20 percent
increase is also attainable.

If the statements above are true, which one of the following must also be true?

(A) It is only Jinx’s production structure that makes it possible for Jinx to survive
the transformation of the market.
(B) Jinx will not go bankrupt if it achieves a productivity increase of 20 percent
over the next two years.
(C) If the gaming market had not been transformed, Jinx would have required
no productivity increase in order to avoid bankruptcy.
(D) Because of the transformation of the gaming market, Jinx will achieve a
productivity increase of 10 percent over the next two years.
(E) If a 20 percent productivity increase is unattainable for Jinx, it must go
bankrupt.

gmat.crackverbal.com
CrackVerbal Critical Reasoning Guide|124

6. Massive layoffs are expected to happen if the VP of Finance is promoted as


the CEO of Transcorp. The company is likely to be split if the VP of Operations is
promoted to CEO instead. The VP of Finance and the VP of Operations are the
only two strong contenders from within Transcorp for the CEO position.

Given the statements in the passage, which one of the following statements
must be true?

(A) Transcorp will certainly have to face either massive layoffs or a split.
(B) If massive layoffs happen at Transcorp, the VP of Finance would have be-
come its CEO.
(C) If massive layoffs do not happen, then the VP of Operations would have
become the CEO of Transcorp.
(D) If both massive layoffs and a split happen, then the CEO of Transcorp would
be neither the VP of Finance nor the VP of Operations.
(E) It is possible that Transcorp will neither see massive layoffs nor be split.

gmat.crackverbal.com
CrackVerbal Critical Reasoning Guide|125

7. Retroviral injections prove to be the only effective means of treating some


serious diseases. Nevertheless, it cannot be stated that retroviral injections
are always beneficial for human health, since they are known to reduce
immune system activity in some cases, and persons with reduced immune
system activity are likely to be afflicted by various infectious diseases.

Which of the following statements can be properly inferred from the passage
above?

(A) Retroviral injections are the major cause of reduced immune system activi-
ty.
(B) Treatment of some serious diseases always results in reduced immune
system activity.
(C) Retroviral injections are not beneficial for human health.
(D) People who contract serious diseases will be afflicted by various infectious
diseases.
(E) Retroviral injections are the only effective means of treating some infec-
tious diseases.

gmat.crackverbal.com
CrackVerbal Critical Reasoning Guide|126

8. The unemployment rate in Prelandia fell from 8 percent in 1881 to 7 percent


in 1886. It cannot, however, be properly concluded from these statistics that
the number of unemployed people in 1886 Prelandia was lower than it had
been in 1881 because ________.

(A) The average time that employees stay in any one job dropped during the
period 1881 to 1886
(B) The total available work force, including those with and without employ-
ment, increased between 1881 and 1886
(C) In some mid-western industrial states, the unemployment rate was much
higher in 1886 than it had been in 1881
(D) Many of the high-paying industrial jobs available in 1881 were replaced by
low-wage service jobs in 1886, resulting in displacements of hundreds of
thousands of workers
(E) help-wanted advertisements increased between 1881 and 1886

gmat.crackverbal.com
CrackVerbal Critical Reasoning Guide|127

9. The cost of producing nuclear submarines in Atlantis is 10 percent less than


the cost of producing nuclear submarines in Utopia. Even after transportation
fees and tariff charges are added, it is still cheaper for a military establishment
to import nuclear submarines from Atlantis to Utopia than to produce nuclear
submarines in Utopia.

The statements above, if true, best support which of the following assertions?

(A) Labor costs in Atlantis are 10 percent below those in Utopia.


(B) Importing nuclear submarines from Atlantis to Utopia will eliminate 10
percent of the manufacturing jobs in Utopia.
(C) The tariff on a nuclear submarine imported from Atlantis to Utopia is less
than 10 percent of the cost of manufacturing the nuclear submarine in Utopia.
(D) The fee for transporting a nuclear submarine from Atlantis to Utopia is more
than 10 percent of the cost of manufacturing the nuclear submarine in Atlantis.
(E) It takes 10 percent less time to manufacture a nuclear submarine in Atlantis
than it does in Utopia.

gmat.crackverbal.com
CrackVerbal Critical Reasoning Guide|128

10. It has been found that excess diclofenac effluents from pharmaceutical
laboratories have been found to be present in surrounding lakes and rivers.
Taken in larger-than-advised quantities, these diclofenac effluents could have
serious health effects, but they are present in quantities far too low to cause
any physiological response in people who drink the water or bathe in it.
Nevertheless, medical experts contend that eliminating these trace amounts
from the water will have public health benefits, since _________.
(A) Some of the diclofenac effluents found in lakes and rivers, even in large
quantities, are harmless to humans
(B) disease-causing bacteria exposed to low concentrations of certain diclo-
fenac effluents can become resistant to them
(C) The side effects of medicines containing diclofenac have not yet been fully
explored
(D) Most diclofenac effluents that reach lakes or rivers rapidly break down into
harmless substances
(E) Some of the diclofenac effluents found in lakes and rivers can counteract
possible harmful effects of other such medical pollutants found there.

gmat.crackverbal.com
CrackVerbal Critical Reasoning Guide|129

ANSWER EXPLANATIONS

1. Answer: B

The fact is that baking compensates for lack of sunlight by boosting the brain’s
levels of serotonin. This implies that lack of sunlight results in lowered seroto-
nin. Therefore option B is relevant. Option D is close, but it talks about antide-
pressants in general, whereas the question mentions only ’some antidepres-
sants’.

(A) The question doesn’t give you information about anti-depressants.


(B) The given option is 100% true based on information given.
(C) There is no information to support the season in which people will be
depressed.
(D) It need not be only serotonin. It could be any neurotransmitter.
(E) It need not be “raising”. It could even happen by lowering the levels.

2. Answer: D

The correct answer choice must be 100% true given the information in the
passage.

(A) Is this the simplest way? We don’t know, can’t say.


(B) Again, we don’t know; can’t say this for sure.
(C) Is this the best way? We are not sure.
(D) The mayor was benefitted by his action. The use of the word ’sometimes’
makes this answer choice hit the balance between ’overly specific’ and ’overly
generic’.
(E) Although this can be one of the drivers, is this the primary driver? We are
not sure.

gmat.crackverbal.com
CrackVerbal Critical Reasoning Guide|130

3. Answer: B

(A) We cannot make a generalization about all poets. This is an extreme answer
choice.
(B) This option is 100% true based on the information given.
(C) This is extreme as it talks about an absolute “every” well-educated person.
This cannot be substantiated by the text.
(D) This is a generic statement not supported by the facts of the passage.
(E) This is a generic statement not supported by the facts of the passage.

4. Answer: E

(A) It indicates higher water consumption.


(B) This option doesn’t indicate whether the water consumption will exactly be
higher.
(C) It indicates higher water consumption.
(D) There is no information to support this.
(E) The argument suggests that the consumption of recycled water will match
fresh water consumption in 2015. Let’s look at the facts. We know that current-
ly fresh water consumption = 2X while recycled water consumption = X. We
also know that in 2015, fresh water consumption will be higher than in the
previous year and fresh water consumption will match recycled water con-
sumption. Therefore total water consumptions will be > 2X+2X which will
therefore definitely be more than 4/3 times greater; So, options A and C are
incorrect. But we do not know for sure whether it will be twice that of the
previous year. So, option B is also incorrect. We don’t have information to
support what option D suggests. Option E however is correct as the previous
year’s recycled water consumption is X and that of 2015 will be greater than
2X. Option E is therefore the apt response.

gmat.crackverbal.com
CrackVerbal Critical Reasoning Guide|131

5. Answer: E

(A) The argument doesn’t provide information to convey that production struc-
ture is the only reason for its survival.
(B) There is information to conclude that Jinx could go bankrupt.
(C) We are not completely sure of this.
(D) The given information doesn’t support this option.
(E) From the facts provided we know that if Jinx does not increase productivity
by 10% it will become bankrupt; we also know that if it is able to increase
productivity to 10% it has the ability to attain 20% increase in productivity.
Therefore, it can be inferred that if a 20% profitability is not attainable Jinx will
become bankrupt. Option E is therefore true.

6. Answer: E

(A) This is not true —the two VPs are the only strong contenders for the CEO
position from within Transcorp. What if an external person is hired as the CEO?
Then the company may not have to face either possibility.
(B) The VP of Finance may cause massive layoffs, but he is not the only reason
for massive layoffs, i.e., X causes Y but just because Y happens, we cannot
assume that X has happened; another cause, Z, could also cause Y.
(C) If massive layoffs do not happen, the only thing we can infer is that the VP
of Finance did not become the CEO of Transcorp; this doesn’t automatically
mean that the VP of Operations became the CEO. The possibility of a third
person exists.
(D) There is no evidence in the passage to support this. We can’t infer what will
happen if both massive layoffs and a split happen.
(E) The possibility exists that a third person may become the CEO in which case
neither massive layoffs nor a split would happen. Therefore, this is the correct
answer.

gmat.crackverbal.com
CrackVerbal Critical Reasoning Guide|132

7. Answer: B

(A)We don’t know whether retroviral infections are the major cause of reduced
immune system activity.
(B) The facts are that retroviral injections are the only effective treatment
methods for certain diseases. It also states that this use reduces immune func-
tion. It can therefore be inferred that treatment for some diseases always leads
to reduced immune function. Option B is therefore the correct response.
(C) This is a blanket-statement. There could be cases in which retroviral infec-
tions prove beneficial.
(D)This talks about all serious diseases.
(E) We don’t have proof about the treatment of some diseases.

8. Answer: B

(A) This does not contribute to the argument as it does not talk about the
percentage of unemployment and the number of unemployed.
(B) The argument suggests that the decrease in percentage of unemployment
need not necessarily imply a decrease in total number of unemployed people.
The only reasoning that would support this is that provided by option B. If the
total workforce increases between 1881 and 1886, the number of unemployed
in 1886 would be higher —even though the percentage is lower.
(C) The argument concerns the decrease in unemployment.
(D) This doesn’t contribute to the fact that there has been a decrease in the
percentage of unemployment.
(E) The argument is not concerned about the increase in the advertisements.

gmat.crackverbal.com
CrackVerbal Critical Reasoning Guide|133

9. Answer: C

(A) We are not concerned with labor costs.


(B) There is no information to support this option.
(C) The fact is that the cost of producing nuclear submarines is 10% less in
Atlantis than in Utopia, even with transportation and other costs included. The
cost to import nuclear submarine to Utopia from Atlantis will still be cheaper.
Therefore, the import related tariff is less than 10% of the cost of manufactur-
ing in Utopia. This is expressed in option C, which is the correct answer.
(D) This doesn’t relate to the reduction in the cost.
(E) This option talks about time. We are concerned about the costs.

10. Answer: B

(A) It doesn’t substantiate the fact that removal of this vaccine can cause good
to human beings.
(B) The argument concludes that removing these retroviral vaccines from the
water bodies will be useful to the human community. We are expected to pick
an answer choice that provides substantiation for this reasoning. Only option B
discusses a scenario in which NOT clearing the retroviral vaccines could be
harmful.
(C) We are not worried about whether the side-effects have been fully ex-
plored.
(D) This might be a contender. However, we are more worried about the
removal of these chemicals.
(E) We are not worried about other medical pollutants.

gmat.crackverbal.com
CrackVerbal Critical Reasoning Guide|134

TYPE 2: RESOLVE THE PARADOX

A paradox is an inconsistent statement or argument, which contradicts itself.


For instance,

I always tell lies.

The following statement is false. The previous statement is true.

’Resolve the paradox’ questions test your ability to understand and process
information without altering it. In such questions, the question gives facts that
appear to contradict each other. Your first task is to identify that para-
dox/discrepancy. Your second task is to choose the answer choice that best
explains how the two facts actually do not contradict one another. The chal-
lenge is that you need to do this without changing either part of the paradox. In
these questions, there are no conclusions —there are only inconsistent prem-
ises.

For example, if X happens, Y should happen. However, the question will tell you
that X happened but Y didn’t happen. You need to explain why. You would need
to understand both X and Y to resolve this paradox correctly. Words of contra-
diction are often used in such questions. For example: but, however, yet, alt-
hough, paradoxically, surprisingly, etc.

Some other terms generally used to describe this problem type are-
• Paradox
• Discrepancy
• Contradiction
• Conflict
• Puzzle
• Resolve
• Explain
• Reconcile
• Justify
gmat.crackverbal.com
CrackVerbal Critical Reasoning Guide|135

Strategy for resolve the paradox questions

It is best to stick to only those answer choices that clearly discuss/point out the
discrepancy between X and Y. Eliminate answer choices that do not do this.

No matter how absurd / ridiculous /counter-intuitive the answer may sound,


you need to accept it as true!

Illustrative example

Many people think that genetic manipulation of fish is being aimed at develop-
ing bigger and bulkier species so that the mass per fish is higher. On the other
hand other fish breeders have in fact concentrated on discovering or producing
dwarf varieties, which are roughly half as bulk as normal varieties are.

Which of the following would, if true, most help to explain the strategy of the
fish breeders referred to above?

(A) Fish varieties are used as food by some and as show pieces in aquariums by
others.
(B) The wholesale prices of a given fish variety decreases as the supply of it
increases.
(C) Fish once bred exclusively for human consumption are often used for
decorative purposes now.
(D) Smaller fish are less vulnerable to capture by predators and hence the
chance of survival is huge.
(E) Nations with large industrial sectors tend to consume more processed fish.

The first step to process the above argument is to find out the apparent contra-
diction. The contradiction is that while people are thinking about growing
bigger fish, many people are now concentrating on breeding smaller fish
varieties.

gmat.crackverbal.com
CrackVerbal Critical Reasoning Guide|136

We now need to find out an option that bridges the apparent contradiction.

(A) This option does not bridge the controversy.


(B) This may be true in reality but does not tell us why people focus on growing
smaller breeds.
(C) Again, this talks about how this can be used but doesn’t tell us why the
smaller fish should be bred.
(D) This option tells you why when people think about big fish, many people
are concentrating on small fish.
(E) We are not worried about consumption of the fish.

gmat.crackverbal.com
CrackVerbal Critical Reasoning Guide|137

PRACTICE QUESTIONS: RESOLVE THE PARADOX QUESTIONS

1. A recent report that artichokes contain a cholesterol-lowering compound


called Luteolin apparently had little effect on consumers. Very few consumers
decided to start buying artichokes as a result of the report. However, sales of
artichokes in grocery stores increased sharply in June, a month after the
report was announced.

Which of the following, if true, best explains the reason for the apparent dis-
crepancy described above?

(A) In June, many artichoke suppliers supplied artichokes to grocers at half the
price, thus resulting in much cheaper artichoke availability in large quantities
to consumers.
(B) The growing number of food-related reports since January caused consum-
ers to lose interest in such news.
(C) The report was aired on the radio and also appeared in newspapers.
(D) Though Luteolin is found in many vegetables, the report did not mention
any vegetable other than the artichoke.
(E) In May, another report citing that artichokes may be sprayed with pesticides
containing cancer-causing chemicals was released a week after the first report.

gmat.crackverbal.com
CrackVerbal Critical Reasoning Guide|138

2. After replacing his old gas water heater with a new pilotless gas water heater
that is rated as highly efficient, Tommy’s gas bills increased.

Each of the following, if true, contributes to an explanation of the increase


mentioned above EXCEPT:

(A) The new water heater uses a smaller percentage of the gas used by Tommy’s
household than did the old one.
(B) Shortly after the new water heater was installed, Tommy’s extended family
came to live with him, doubling the size of the household.
(C) Tommy bought and started using a gas dryer instead of going to the Laun-
dromat as before after he replaced his water heater.
(D) Tommy’s utility company raised the rates for gas consumption following
installation of the new water heater.
(E) Tommy replaced his water heater just before winter in time to meet the
increased consumption of heated water during the winter months.

gmat.crackverbal.com
CrackVerbal Critical Reasoning Guide|139

3. Woodsville’s bear population has been declining in recent years, primarily


because of hunting. Bears prey heavily on a species of freshwater fish that is
highly valued as food by the residents of Woodsville, who had hoped that the
decline in the bear population would lead to an increase in the number of these
fish available for human consumption. Yet the population of this fish species
has also declined, even though the annual number caught for human consump-
tion has not increased.

Which of the following, if true, most helps to explain the decline in the popula-
tion of the fish species?

(A) Over the last few years, Woodsville’s commercial fishing enterprises have
increased the number of fishing boats they use.
(B) During Woodsville’s dry season, ditches dug by bears remain filled with
water long enough to provide a safe place for the eggs of this fish species to
hatch.
(C) Many Woodsvilleers who hunt bears do so because of the high market price
of bear skins, not because of the threat bears pose to the fish population.
(D) The decline in the bear population has meant that fishers can work in some
parts of lakes and rivers that were formerly too dangerous.
(E) A wild animal epidemic that could also affect bears has been raging in
several neighboring counties of Woodsville.

gmat.crackverbal.com
CrackVerbal Critical Reasoning Guide|140

4. Dried grass clippings mixed into garden soil gradually decompose, providing
nutrients for beneficial soil bacteria. This results in better-than-average plant
growth. Yet, mixing fresh grass clippings into garden soil usually causes
poorer-than-average plant growth.

Which one of the following, if true, most helps to explain the difference in plant
growth described above?

(A) The number of beneficial soil bacteria increases whenever any kind of plant
material is mixed into garden soil.
(B) Nutrients released by dried grass clippings are immediately available to
beneficial soil bacteria.
(C) Some dried grass clippings retain nutrients originally derived from commer-
cial lawn fertilizers, and thus provide additional enrichment to the soil.
(D) Fresh grass clippings mixed into soil decompose rapidly, generating high
levels of heat that kill beneficial soil bacteria.
(E) When a mix of fresh and dried grass clippings is mixed into garden soil,
plant growth often decreases.

gmat.crackverbal.com
CrackVerbal Critical Reasoning Guide|141

5. During the seventeenth century, the English Academy of Science was a major
financial sponsor of research in physics and chemistry in England. Financial
support by private individuals had decreased dramatically by this time. Be-
cause the Academy discouraged innovation in the sciences, there was little
innovation in seventeenth century England in the field of chemistry. Yet a
remarkable degree of innovation was seen in the field of physics during the
same period in England.

Which one of the following, if true, most helps to explain the difference be-
tween the level of innovation in the fields of physics and chemistry during the
seventeenth century in England?

(A) The Academy offered more support to physics than to chemistry during
seventeenth century England.
(B) The Academy in seventeenth century England financially supported a great-
er number of physicists than chemists, but individual chemists received more
support, on average, than individual physicists did.
(C) Since research in chemistry required the purchase of expensive chemicals
and equipment and physics did not, there were far more unsponsored physi-
cists than unsponsored chemists in England during the seventeenth century.
(D) Very few of the scientists in England in the seventeenth century who spe-
cialized in research in chemistry also specialized in research in physics.
(E) Although the Academy was the primary sponsor of research in chemistry and
physics, the total amount of financial support that English chemists and physi-
cists received from sponsors declined during the seventeenth century.

gmat.crackverbal.com
CrackVerbal Critical Reasoning Guide|142

6. For a long time, Monterey beach authorities have had an active feeding
program for the otters of Monterey bay. Many otters get as much as half of their
daily food requirement each day from the fish and other nourishments that the
beach authorities provide. However, the life expectancy of Monterey bay otters
has become considerably less than that of wild otters.

Which of the following, if true, most helps to explain this discrepancy?

(A) The fish and nourishments provided by Monterey beach authorities have not
been tested by a central animal health authority.
(B) There has been an increase in recent years in the number of sharks in and
around Monterey bay.
(C) The survival instincts and skills of Monterey bay otters have decreased as a
result of their dependence on human feeding.
(D) Monterey bay otters have become overly friendly with humans because of
their interaction with the beach authorities.
(E) The fish given to the otters at the beach are the same types of fish that
otters typically catch in the wild.

gmat.crackverbal.com
CrackVerbal Critical Reasoning Guide|143

ANSWER EXPLANATIONS

1. Answer: A

(A) If the price of artichokes was reduced and its availability increased in June,
more consumers would have decided to buy them. Thus, the right answer is A.
(B) This explains why consumers did not change their artichoke-buying habits
in May and not why sales increased sharply in June.
(C) We are not worried about how consumers may have heard about the report.
(D) Vegetables other than artichokes aren’t part of the discussion.
(E) It doesn’t give information on how sales increased in June.

2. Answer: A

The paradox is between higher efficiency of the water heater and the increased
bill amount after its purchase. An increase in consumption or an increase in the
price of gas might explain this. Options B, C, D, and E give a possible explana-
tion. Option A does not.

(A) This not related to the argument.


(B) This tells you the reason why there was a raise in the bill.
(C) Again, this explains why there was increase in the bill.
(D) This explains why there was increase in the bill.
(E) This option is capable of explaining why there was an increase in the bill.

gmat.crackverbal.com
CrackVerbal Critical Reasoning Guide|144

3. Answer: B

(A) It doesn’t link the reductions in bear population and fish population.
(B) The contrasting facts are that even though the bear population decreased,
the fish population decreased and human consumption of fish did not change.
An explanation is needed to link the reduction of bear population with that of
the fish population. Only option B provides a fact that suggests that the ab-
sence of bears could have resulted in a decrease in the population of fish.
Option B is therefore correct.
(C) This option doesn’t explain why the fish population decreased.
(D) This option doesn’t explain why the fish population decreased.
(E) It doesn’t link the reduction in bear population and fish population.

4. Answer: D

(A) This option doesn’t tell us why mixing fresh grass isn’t beneficial to the
bacteria.
(B) This says that it might be beneficial to the bacteria whereas we want an
option that tells us that it isn’t beneficial.
(C) Again, this option doesn’t tell us why mixing fresh grass isn’t beneficial to
the bacteria.
(D) The contrasting facts are that while dried grass is great for beneficial bacte-
ria, mixing fresh grass isn’t. The correct answer should address this discrepancy
and possibly introduce evidence that shows that fresh clippings are harmful to
beneficial bacteria. Option D addresses this and explains the paradox.
(E) Again, this option does not explain why fresh grass isn’t beneficial.

gmat.crackverbal.com
CrackVerbal Critical Reasoning Guide|145

5. Answer: C

(A) This could be true. However, this doesn’t tell us why there were more num-
ber of innovations in physics.
(B) This option talks about both sides. According to the logic, there should also
have given equal number of innovations in chemistry.
(C) The contrasting facts are as follows. The Academy of Science was the major
financial sponsor (for chemistry and physics) and it discouraged innovation. Yet
physics showed a high level of innovation when compared to chemistry. The
explanation could lie in the fact that these innovations were achieved by
non-sponsored physicists. Option C helps explain this paradox - showing how
physics had high levels of innovation while chemistry didn’t! The other options
do nothing to address the apparent contradiction.
(D) Again, this doesn’t tell us why there were more number of innovations in
physics.
(E) This option doesn’t tell us why the number of innovations were high in
physics.

6. Answer: C

(A) Just because the food has not yet been tested, it does not necessarily mean
that the food does not meet the health requirements
(B) Do sharks feed on otters? The information is insufficient.
(C) The discrepancy here is that Monterey bay otters have a lower life expec-
tancy than that of wild otters despite being fed by beach authorities. The
explanation lies in the change in behavior of the otters that are fed by the
program. Only option C gives this perspective. If the otters have lost their
survival skills, their life expectancy would also drop.
(D) So what? Has this friendliness led to any adverse effects? The information is
insufficient.
(E) This strengthens the paradox and doesn’t help explain it.

gmat.crackverbal.com
CrackVerbal Critical Reasoning Guide|146

gmat.crackverbal.com
CrackVerbal Critical Reasoning Guide|147

7. Structure-based questions

• Type 1: Bold-Faced Questions


• Step-by-Step Approach for Bold-Faced Questions
• Practice Questions: Bold-Faced
• Type 2 : Parallel or Mimic the Reasoning
• Practice Questions: Mimic the Reasoning

gmat.crackverbal.com
CrackVerbal Critical Reasoning Guide|148

gmat.crackverbal.com
CrackVerbal Critical Reasoning Guide|149

As the name suggests, structure-based CR questions require the test-taker


to be able to understand the structure of the argument at the most fundamen-
tal level, i.e., you need to identify which question statements indicate the
evidence, conclusion, counter-conclusion, etc., and the relationships between
them.

TYPE 1: BOLD-FACED QUESTIONS

Bold-faced questions require you to use the same skills to identify argument
structure that we have covered in assumption based questions. In such ques-
tions, 2 sentences in the argument statement will be in bold. You would be
required to identify the logical relationship between these 2 sentences.

Bold-faced questions are tough because of the following 2 reasons:

Trap 1: The actual conclusion/point of the author is hidden by the coun-


ter-argument provided. So this makes the argument look like it has 2 conclu-
sions —each contradictory to the other. The argument could also have an
intermediate conclusion in addition to the final or main conclusion
—identifying which is which is tricky.

Consider this example:

Students who do well on the GMAT typically drink Red Bull during the break.
There are a lot of students who have done well while taking Red Bull. However,
recent studies have shown that taking Red Bull can actually be bad for concen-
tration and that the students who took Red Bull and scored high, could have
scored higher if they had not taken it. Hence, it is incorrect to believe that Red
Bull can help your performance on the GMAT.

gmat.crackverbal.com
CrackVerbal Critical Reasoning Guide|150

So what is the main argument here?

The argument’s main perspective is that “Red bull may not help you perform
better”. Let’s look at the other components of the argument. The sentence
“however, recent studies…” provides support to the main conclusion: let’s call
this the premise. The first sentence of the stimulus gives a perspective that is
contradicted by the main conclusion. This is a fact statement (the opinion that
’Red Bull is good’ is implied); let’s call this the counter-premise since it is
counter to the main conclusion (and helps the counter —conclusion). Therefore
the first bold-face is a counter-premise and the second bold-face is the main
conclusion. That was an example of how the structure can get complicated.

Remember, bold-faced questions can only test you on 5 things:

(A) Conclusion
(B)Counter conclusion
(C)Intermediate conclusion
(D)Premise/considerations that help the conclusion
(E) Premises/considerations that help the counter conclusion

Yet, it can complicate another aspect of the question: the answers.

Trap 2: The answer choices use phrase/words that make it sound confusing. The
correct answer option for the argument above would be worded as: “The first is
a premise presented to support a position against an argument that is supported
by the author; the second presents an objection to that position”

Imagine having to read and choose the correct one out of five such answer
choices in just over a minute!

gmat.crackverbal.com
CrackVerbal Critical Reasoning Guide|151

STEP - BY-STEP APPROACH FOR BOLD-FACED QUESTIONS

Bold-faced questions are easy to tackle if you understand and apply the fol-
lowing techniques well:

1. Identify the conclusion. Ask yourself what the conclusion is, what the author
is trying to prove, or what the author’s main point is.
2. Vertically scan and eliminate obvious errors in the answer choices for the 1st
bold-faced statement before you look at the options for the 2nd bold-faced
statement.
3. Search for argument indicators to determine the relationship between
evidence and conclusion.

Remember to use the one-sentence technique to identify and confirm the main
argument/position in these questions.

Illustrative example

Fisherman: Many people blame fishermen alone for decline in the Greenrock-
lake’s angelfish population over the past ten years. Yet clearly, another variety
of fish called the salmon fish has also played an important role in this decline.
In the past ten years, the lake’s protected salmon fish population has risen
sharply and examination of salmon fish found dead in the lake during the
fishing season shows that a number of them had recently fed on angelfish.

In the fisherman’s argument, the portion in boldface plays which of the follow-
ing roles?

(A) It is the main conclusion of the argument.


(B) It is a finding that the argument seeks to explain.
(C) It is an explanation that the argument concludes is correct.
(D) It provides evidence in support of the main conclusion of the argument.
(E) It introduces a judgement that the argument opposes.

gmat.crackverbal.com
CrackVerbal Critical Reasoning Guide|152

What do you understand from the argument?

The main conclusion of the argument is that salmon fish have played an im-
portant role in the decline of angelfish population. The bolded phrase is con-
troversial to this statement.

(A) This is not the main conclusion.


(B) The argument doesn’t explain the bolded phrase.
(C) The bolded phrase is not an explanation.
(D) This is not an evidence.
(E) This is very relevant to the reasoning mentioned above.

gmat.crackverbal.com
CrackVerbal Critical Reasoning Guide|153

TERMINOLOGY COMMONLY USED IN BOLD-FACED QUESTIONS


Premise

• Evidence/finding/illustration • Reason
• Proof supposition • Situation
• Data • Circumstance
• Information • Reasoning/line of reasoning
• Research • Generalization
• Consideration • Observation
• Pattern • Support
• Explanation • Example
• Justification • Phenomenon

Conclusion

• Main point • Forecast


• Author’s position • Claim
• Assertion • Stand
• Judgment • Prediction
• Opinion • Hypothesis
• Suggestion • Position
• View • Stance
• Idea • Point
• Belief • Implication
• Proposal • Contention
• Warning • Consequence

gmat.crackverbal.com
CrackVerbal Critical Reasoning Guide|154

PRACTICE QUESTIONS: BOLD-FACED TYPE

1. The government has recently been severely criticized for its decision to
block access to a few websites on which malicious information and photo-
graphs were being posted. The government has responded to this criticism by
stating that while it believes in freedom of speech and expression, this partic-
ular instance was an emergency and in such situations, the source of the
problem must be cut off as soon as possible.

In the argument the two portions in boldface play which of the following roles?

(A) The first describes a reaction to an action, and the second describes an
action taken in response to this reaction.
(B) The first provides the criticism of an action and the second provides justifi-
cation for the necessity of taking that action.
(C) The first is a criticism that the argument disagrees with, and the second is
the point of view that the criticism supports.
(D) The first is the point of view of a group of people, and the second attacks
this point of view.
(E) The first provides a counterpoint to the argument’s conclusion, and the
second is that conclusion.

gmat.crackverbal.com
CrackVerbal Critical Reasoning Guide|155

2. Geologist: The element celerium, which originates as a mineral ore known as


bastnasite, is used as a capacitor in computers and cell phones. Most of the
world’s supply of cerleium is exported by India, Australia, and Japan. However,
a small percentage of celerium supply comes from poorly governed or con-
flict-torn regions of Africa and is mined using objectionable practices such as
child labor. A group of scientists has begun working on a method that will be
able to detect chemical signatures that can distinguish cerleium samples
according to their location of origin. Their technique needs to be further
developed into a standardized and widely reproducible method of testing. If
such testing becomes standard use among manufacturers and suppliers, it
would allow companies and consumers to prevent human rights abuse
through the marketplace.

In the geologist’s argument, the two portions in boldface play which of the
following roles?

(A) The first provides evidence in support of the conclusion of the argument;
the second acknowledges a consideration that weighs against that conclusion.
(B) The first is information advanced in support of the conclusion of the argu-
ment; the second is that conclusion.
(C) The first acknowledges a consideration that weighs against the conclusion
of the argument; the second provides evidence in support of that conclusion.
(D) The first presents a circumstance for which the geologist offers an explana-
tion; the second is part of that explanation.
(E) The first acknowledges a consideration that weighs against the conclusion
of the argument; the second is that conclusion.

gmat.crackverbal.com
CrackVerbal Critical Reasoning Guide|156

3. Music creators have been strongly influenced by the demands of devout


music fans who have historically purchased the majority of music labels. Nor-
mally, devoted fans crave sound complexity and lyrical genius; thus, most
creators continue to produce music with complex lyrics and intricate instru-
ment mixes. However, this is not in the industry’s best interest. The devoted
music listener market has become stagnant and will not expand in the near
future. To infuse new life into the music industry, creators must simplify the
form and lyrics of their music. By doing so, they can attract new listeners and
influence them to join the ranks of devout fans.

In the argument, the two portions in boldface play which of the following roles?

(A) The first is a statement of fact that supports the author’s position; the
second is a consideration that weighs against the author’s position.
(B) The first is a prediction that the author believes should not hold in this case;
the second is an assumption that weighs against the author’s position.
(C) The first is a statement of fact that contradicts the author’s position; the
second is the author’s position.
(D) The first is a situation that the author believes to be true; the second offers
evidence to explain this situation.
(E) The first is a situation that the author argues should not continue; the se-
cond provides evidence that supports the author’s position.

gmat.crackverbal.com
CrackVerbal Critical Reasoning Guide|157

4. After the tyrannical reign of dictatorship collapsed in Libya, some hoped that
newfound freedom would encourage Libyans to multiply, but as a result of
dislocation and insecurity, the Libyan population continues to dwindle at the
rate of 2 million a year. The government proposes to address the problem with
a wide range of financial incentives, along with investments in improved health
care, road safety and the like. These are positive measures, but they have been
tried before, to little avail. A better plan to reverse the population decline is to
improve the country’s governance in both the public and the private sphere. If a
greater part of the population participated in important decisions and
shared in the country’s wealth, then larger families would result. In addition,
if corruption and greed among the elite were curbed, public health would
improve and average life expectancy would increase.

The two boldfaced statements serve what function in the argument above?

(A) The first is a premise that supports a proposal; the second is that proposal.
(B) The first is the main point of the argument; the second is a premise that
supports the first.
(C) The first is a premise that contradicts the main point made by the argument;
the second is the main point of the argument.
(D) The first is a premise that challenges the action that the argument opposes;
the second is a premise that supports the main claim.
(E) The first is a conclusion that the argument endorses; the second is a premise
that opposes that conclusion.

gmat.crackverbal.com
CrackVerbal Critical Reasoning Guide|158

5. Any HIV drug that shows positive results in early laboratory tests can gener-
ate a great deal of interest from AIDS patients. Because of this interest, and
the desire to make a profit early in the development cycle, many pharmaceu-
tical companies rush the new drug to market as quickly as possible. But posi-
tive early results are often misleading, meaning that the effort to bring the
drug to market was largely wasted. Thus, a better strategy to maximize
long-term profit from a new HIV drug will be to bring the drug to market only
after its positive effects are more thoroughly tested and established.

In the argument above, the two portions in boldface play which of the follow-
ing roles?

(A) The first is a consideration that has been used to justify pursuing a goal that
the argument rejects; the second presents a course of action that has been
adopted in pursuit of that goal.
(B) The first is a consideration raised to support the strategy that the argument
recommends; the second presents that strategy.
(C) The first is a consideration raised to explain the appeal of a certain strategy;
the second presents an alternative strategy.
(D) The first is an assumption, rejected by the argument, that has been used to
justify a course of action; the second presents that course of action.
(E) The first is a consideration that has been raised to argue that a certain
strategy is counterproductive; the second presents an alternative strategy.

gmat.crackverbal.com
CrackVerbal Critical Reasoning Guide|159

ANSWER EXPLANATIONS
1. Answer: B

(A) The second boldface is not an action taken in response to a reaction.


(B) The first boldface text discusses a criticism made of the government. The
second is a justification made by the government against this criticism. Only
option B portrays this relationship.
(C) The second boldface is not the point of view that the criticism supports.
(D) The first is not a point of view and the second is not an attack.
(E) The first is not a counterpoint to the argument’s conclusion.

2. Answer: B

(A) The first is not evidence.


(B) The main conclusion is that such testing could help sanction human rights
abuses; this is the second boldface. The first boldface is information that the
geologist provides in support of the conclusion. Therefore option B is the most
apt response.
(C) The first doesn’t acknowledge any consideration.
(D) The second is not a part of any explanation.
(E) The first and the second are not controversial to each other.

gmat.crackverbal.com
CrackVerbal Critical Reasoning Guide|160

3. Answer: E

(A)The second is evidence and is not a consideration.


(B) The second is evidence and not a position.
(C) The second is evidence and not an assumption
(D) The author doesn’t believe this to be true. The author wants to stop the
process.
(E) The author suggests that intricate and complex music being produced is
counter intuitive for the music industry. The author’s opinion is that this trend
must be changed to infuse new life into the market. Therefore, the first bold-
face signifies a trend that the author feels must stop; the second boldface is
supporting evidence that corroborates with the author’s opinion. The most apt
response is option E.

4. Answer: D

(A) The first doesn’t support the proposal.


(B) The first is not the main point of the argument.
(C) The second is not the main point.
(D) The argument concludes that improving the country’s governance in both
public and private spheres would be a better plan than what’s being tried. The
first boldface is a statement that indicates the non-viability of the previous
actions to increase population rate. The second boldface supports the main
conclusion. Therefore, the apt response is option D.
(E) The first is not the conclusion.

gmat.crackverbal.com
CrackVerbal Critical Reasoning Guide|161

5. Answer: C

(A)The second is not a course of action.


(B) The second is an alternative strategy.
(C) The argument suggests an alternate strategy —it points out the ineffectual-
ity of the current strategy. The first boldface gives an explanation for why the
previous strategy is followed. The second gives an alternate strategy. There-
fore, the apt response is option C.
(D) The second is not a course of action.
(E) The first doesn’t tell you that something is counterproductive.

gmat.crackverbal.com
CrackVerbal Critical Reasoning Guide|162

TYPE 2: PARALLEL OR MIMIC THE REASONING

Such questions ask you to select a situation that best exemplifies the main
point (the conclusion) of a given argument. These involve the use of logic as
they bring an analogy from ’outside’ of what is stated in the stimulus. On such
questions, a better approach would be to read the answer choices and apply a
process of elimination, rather than select the right choice directly.

Pre-phrase the actual answer by trying to reword the argument in an X & Y


format where you can substitute X & Y with nouns.

For instance, let us look at this argument:

A car that has been tuned by the ’Motor-Head Company’ will have very few
cases of breakdowns and its fuel economy figures will be higher than the
industry average. John’s car has had very few cases of breakdowns and has a
fuel economy rating that is higher than the industry average. Therefore, John
must have gotten his car tuned at the ’Motor-Head Company’.

How can we get to the bare bone structure of this argument?

X = Car gets tuned at the ’Motor-Head Company’

Y = Car has very few cases of breakdowns and its fuel economy figures is higher
than industry average

X happens, then Y happens. The given argument says: Y has happened; there-
fore, X should have happened.

You can now substitute X and Y with the ideas given in the answer choices and
see if it parallels the reasoning in the question statement.

gmat.crackverbal.com
CrackVerbal Critical Reasoning Guide|163

Illustrative example

The lead fitness instructor of Fit-Life gym insists that weight loss targets are not
met because of a disruption in the exercise routine. In a few months, the weight
loss targets of all its clients were met. The instructor therefore concluded that
the trainers were able to ensure that the clients attended the gym without any
disruption in their schedule.

The flawed pattern of reasoning in the above argument is most similar to that
in which of the following?

(A) A popular diabetologist suggested that the high levels of diabetes was
caused because of over-eating. In a brief time all the members stopped over
eating. The diabetologist was please to conclude that all their diabetes
levels were in control.
(B) A diabetologist insisted that the increase in diabetes levels that their clients
complained of was merely in their imagination. Members were given charts to
track their diabetes levels for the last three months. The diabetologist was
pleased to conclude that all the diabetes levels of clients were under control. It
was merely in their imagination. Members were given charts to track their
diabetes levels for the last three months. The diabetologist was pleased to
conclude that all the diabetes levels of clients were under control.
(C) A team lead at a leading BPO insisted that his team members who filed
complaints had too many different calls to take in a particular day. The team
lead simplified the work flow process and the number of calls per person
reduced. The team lead happily concluded that the working environment
had been improved.
(D)A team lead at a leading BPO insisted that his team members who filed
complaints did not have enough calls to attend in a particular day. Soon there
were no complaints filed. The team lead was pleased to conclude that
every team member now had enough calls to take.
(E) The dietician insisted that the weight gain that team members complained
of was caused by their thinking of food too often. The dietician was happy to
conclude that the weight gain had stopped once the team members reported

gmat.crackverbal.com
CrackVerbal Critical Reasoning Guide|164

The first step in the argument is to convert the given argument into a simpler
format with X and Y. Here is how you can do it.

The argument says X is caused by Y. Also X reduced and this led to the reduc-
tion of Y.

The next step now is to re-write all the given options in a similar format.

(A) X is caused by Y. Y reduced and therefore X reduced. This reverses the


entire argument and hence can be eliminated.
(B) X is caused by Y. Z increased and hence Y reduced. This option introduces
another factor which does not happen in the original argument.
(C) X is caused by Y. Y reduced and hence Z reduced. Therefore it was conclud-
ed that Z increased — This option again introduced another parameter Z and
hence doesn’t match with the given argument.
(D) X is caused by Y. X reduced and hence Y reduced. This is similar to what-
ever is mentioned in the argument and hence is the OA.
(E) X is caused by Y. This option again isn’t in sync with the reasoning given in
the argument. Hence, it can be eliminated.

gmat.crackverbal.com
CrackVerbal Critical Reasoning Guide|165

PRACTICE QUESTIONS: MIMIC THE REASONING QUESTIONS

1. People living in cold climates sometimes buy a winter coat that is stylish but
not warm in order to appear sophisticated. This just goes to prove that people
are sometimes willing to sacrifice comfort or pleasure for the sake of appear-
ances.

The analysis provided for the situation above is most appropriate for which one
of the following situations?

(A) A person sets her thermostat at a low temperature during the winter be-
cause she is concerned about the environmental damage caused by using fossil
fuels to heat her home.
(B) A parent buys a car seat for a young child because it is more colorful and
more comfortable for the child than the other car seats on the market, though
not safer.
(C) A person buys an automobile to commute to work even though public
transportation is quick and reliable.
(D) A couple buys a particular wine even though their favorite wine is less
expensive and better tasting because they think it will impress their dinner
guests.
(E) A couple buys a car that has fewer features because their previous car was
of the same manufacturer, even though better cars were available at the same
price.

gmat.crackverbal.com
CrackVerbal Critical Reasoning Guide|166

2. People who are good at playing football are invariably skilled with their feet.
Jane is a very competent ballet dancer. Therefore, Jane would make a good
football player.

The flawed pattern of reasoning in the argument above is most similar to that
in which one of the following?

(A) All race car drivers have good reflexes. Chris is a champion table tennis
player. Therefore, Chris would make a good racecar driver.
(B) People who write for a living invariably enjoy reading. Julie has been a
published novelist for many years. Therefore, Julie enjoys reading.
(C) People with long legs make good runners. Everyone in Daryl’s family has
long legs. Therefore, Daryl would make a good runner.
(D) The role of Santa Claus in a shopping mall is often played by an experienced
actor. Erwin has played Santa Claus in shopping malls for years. Therefore,
Erwin must be an experienced actor.
(E) Any good skier can learn to ice-skate eventually. Erica is a world-class skier.
Therefore, Erica could learn to ice-skate in a day or two.

gmat.crackverbal.com
CrackVerbal Critical Reasoning Guide|167

3. The freeze imposed on the hiring of teachers in Vargonia puts a stop to any
improvement in education quality in the county. The quality of education in
Vargonia will surely deteriorate in the near future.

The flawed reasoning in which one of the following is most similar to that in the
argument?

(A) Papa Mo’s has stopped serving pepperoni pizza, the most popular item on
its menu. Papa Mo’s sales are certainly going to drop soon.
(B) Given that over 250 years of attempts to prove the Goldbach conjecture
have failed, it will probably never be proved.
(C) Since funding levels for social programs are being frozen, social harmony is
unlikely to improve.
(D) Since there is a storm moving in, the outside temperature cannot rise this
afternoon.Therefore, it must fall.
(E) The starter in Mary’s car gave out weeks ago, and so it is impossible for the
car to start. Therefore, it will not start.

gmat.crackverbal.com
CrackVerbal Critical Reasoning Guide|168

4. It is generally true that honest people are uncomfortable misleading others.


In politics, however, things must often go unsaid or be spun a certain way in
order to keep the greater peace. Therefore, it is reasonable to conclude that
honest people should not get into politics.

Which of the following is most like the passage above in logical structure?

(A) Diabetics should not become pastry chefs, because over-exposure to sugar
will be harmful to their health.
(B) People who love to read should avoid science professions, because science
and the arts are mutually exclusive.
(C) Compulsive liars should avoid becoming religious leaders, because most
religions stress the importance of honesty.
(D) Those who cannot put others’ needs before their own should avoid becom-
ing parents because parenting requires selflessness.
(E) Professional basketball players should avoid playing against amateurs,
because people who are less talented than they are can only weaken their skills

gmat.crackverbal.com
CrackVerbal Critical Reasoning Guide|169

5. Salesperson: When shopping, the average customer, after researching a


product, will not purchase it if it is overpriced. Most music enthusiasts research
entertainment products before buying them. So, there is no point trying to sell
our high-fidelity speaker systems since they are overpriced.

The pattern of reasoning in which one of the following arguments is most


similar to that in the salesperson’s argument above?

(A) Most amateur critics will not review a book if they have reason to believe
they will not enjoy reading it. Nobody who dislikes a certain genre of music
will enjoy reading a book on the history and theory of that genre. Thus, nobody
will review your book on the history and theory of country music, since no
amateur critics enjoy listening to country music.

(B) The union of education sector employees will not endorse a candidate who
does not support their interests. None of our party’s candidates standing for
school board elections support the interests of this union. So none of our
party’s candidates for the school board will be endorsed by the union.

(C) Nobody will tour an island getaway if they believe other islands in the area
have more attractive beaches. Anybody taking a vacation in this stretch of
islands will have researched all of the beaches on all of the islands in the area.
So our tourism agency should improve the public image of our beaches.

(D) Famous actors and actresses will support an organization only if they be-
lieve that their cause is just. Patricia Moore, a famous actress, does not believe
that the cause of Green Thumbs Up is just. Therefore, no famous actors and
actresses will support Green Thumbs Up.

(E) No female guppy will mate with a male guppy if the male is visibly less
healthy than other males in the school. Therefore, a male guppy which has no
visible ill-health symptoms will have no trouble mating.

gmat.crackverbal.com
CrackVerbal Critical Reasoning Guide|170

ANSWER EXPLANATIONS

1. Answer: D

The reasoning is that people prefer appearance over utility. The only option
that mimics this reasoning is option D. C is a close option, but is incorrect
because it doesn’t say that the person bought the automobile because it would
appear better.

(A) There is no preference given to anything here.


(B) There is no preference to aesthetic sense.
(C) There is no preference to aesthetic sense.
(D) This is similar to the reasoning mentioned in the argument
(E) The preference here is given to the manufacturer and not to any feature of
the car.

2. Answer: A

The reasoning is that people who are good at X invariably have quality Y.
Therefore, someone who does Z that involves a form of skill Y will be good at X.
The only option that follows this line of reasoning is option A.

(A) This is similar to the reasoning mentioned in the argument.


(B) People who do X enjoy doing Y. Julie has been good with X and hence she
will be good with Y. This doesn’t really sync with the given argument.
(C) Here we are bringing in the family and this is not in sync with the given
argument.
(D) This option reverses the relationship.
(E) This option doesn’t bring the third factor X.

gmat.crackverbal.com
CrackVerbal Critical Reasoning Guide|171

3. Answer: D

The reasoning is that since X has been stopped, Y will not improve as a result,
and therefore the opposite of Y will happen. D is the only answer choice that
mimics this.

(A) This option has no “Y” indicated.


(B) This is a prediction rather than a causal relationship.
(C) This does not say that the opposite of Y will happen, i.e., that social harmony
will deteriorate
(D) This is similar to the reasoning given in the argument.
(E) This is a simple cause-effect relation. It does not follow the argument pat-
tern.

4. Answer: D

The reasoning here is that a particular field/life style requires X quality. There-
fore, people who lack that quality should not enter that field/life style. Only
option D brings out this relationship precisely.

(A) This doesn’t talk about any particular quality to be possessed.


(B) Again, this doesn’t talk about a particular quality to be possessed.
(C) This option doesn’t sync with the argument given.
(D) This is similar to the reasoning given in the argument.
(E) This talks about reduction of skills. Hence, it is not in sync with the argu-
ment.

gmat.crackverbal.com
CrackVerbal Critical Reasoning Guide|172

5. Answer: B

The argument’s reasoning is that “if X has quality Y (undesirable), X should not
be supported/endorsed/catered to. The only option that follows this line of
reasoning is option B.

(A) This option brings too many factors whereas the given argument doesn’t
introduce that many factors.
(B) This is similar to the reasoning given in the argument.
(C) This brings in the quality of other comparable entities. This doesn’t happen
in the argument.
(D) This option follows the “generalization” pattern. And the same does not
happen in the given argument.
(E) This option doesn’t really tell us that a particular X shouldn’t be endorsed. It
just tells you the difficulty that the X will face because of the unfavorable
quality.

gmat.crackverbal.com
CrackVerbal Critical Reasoning Guide|173

8. Advanced Test-Taking Strategies for Critical Reasoning

gmat.crackverbal.com
CrackVerbal Critical Reasoning Guide|174

gmat.crackverbal.com
CrackVerbal Critical Reasoning Guide|175

Let us now look at some test-taking strategies for critical reasoning

STRATEGY FOR ASSUMPTION-BASED QUESTIONS

1. Try to zone in on the conclusion first while glossing over the premise. Try to
read the premise in detail in the 2nd passage.

2. Do not use negation technique on all assumption questions —or still worse,
on all answer choices. Use it only when you are stuck between the last 2 op-
tions or when you need to validate an answer choice you are not very comfort-
able with.

3. Negation in strengthen and weaken questions will not necessarily yield the
opposite and should be used sparingly.

STRATEGY FOR CONTENT-BASED QUESTIONS

1. In content-based questions, unlike in Quant, you will be given more infor-


mation than you need to make inferences. All of what is given in the argu-
ment statement might not be required to make inferences.

2. On inference questions, the correct answer “must be true”. However, wrong


answer choices “can be true” or “must be false”.

gmat.crackverbal.com
CrackVerbal Critical Reasoning Guide|176

Strategy if you Get Stuck!

1. Take Your Time: Critical reasoning prompts contain logical traps and demand
careful reading. CR questions often take far longer than SC or RC questions do -
two and half minutes to three minutes is not too long for a tough CR question.

2. Draw a Diagram: Some students find it helpful to draw diagrams of the


argument, using letters or pictures to represent entities and their relationships.
Doing so can help untangle complicated arguments.

3. Eliminate Wrong Answers for a Reason: When stuck between two answer
choices, identify the exact reason why either answer choice is incorrect. Look
for distortions and shifts in terms, irrelevant comparisons, extreme language,
and other common traps.

v Access Code is CVFOUNDATION

gmat.crackverbal.com
CrackVerbal Critical Reasoning Guide|177

CR – Mind-Map

gmat.crackverbal.com
CrackVerbal Critical Reasoning Guide|178

gmat.crackverbal.com

Você também pode gostar